UPSC NOVEMBER- 2019 DAILY CURRENT AFFAIRS CLASS TEST - 01 1. ‘Fridays for Future’ recently seen in news is 3. ‘Gadgil Committee Report’ and related to ‘Kasturirangan Committee Report’, (a) A scheme introduced by GOI to involve sometimes seen in the news, are related to students in sports. (a) Constitutional reforms (b) A movement to protest against the lack of (b) Ganga Action Plan action on the climate crisis. (c) Linking of rivers (c) A cancer awareness programme done by (d) Protection of Western Ghats school students. Answer: d) (d) A movement against the global terrorism. Answer: b) 4. Consider the following statements Explanation: regarding UNFCCC: Fridays for Future is a movement that began in 1. It is an international treaty which aims August 2018, after 15 years old Greta Thunberg to reduce greenhouse gas emissions. sat in front of the Swedish parliament every 2. Kyoto protocol was signed under the school day for three weeks, to protest against the framework of UNFCCC. lack of action on the climate crisis. Choose the correct answer using the code given

below: 2. Article 124 of the Indian Constitution deals (a) 1 only with which of the following? (b) 2 only (a) Appointment of Supreme Court Judges (c) Both 1 and 2 (b) Appointment of ad hoc Judges (d) Neither 1 nor 2 (c) Seat of Supreme Court Answer: c) (d) Original jurisdiction of the Supreme Court Explanation: Answer: a) The UNFCCC, signed in 1992 at the United Explanation: Nations Conference on Environment and Article 124 of the Constitution of provides Development also known as the Earth Summit, the matter of appointing judges of the Supreme the Rio Summit or the Rio Conference. It is an Court. international treaty which aims to reduce Though no specific provision exists in the greenhouse gas emissions. So, statement 1 is Constitution for appointing the Chief Justice, true. who as a result, is appointed like the other judges In 1997, third meeting of UNFCCC has happened conventionally, the outgoing CJI recommends (also known as COP3). In this meeting Kyoto the name of the senior-most judge (i.e. by date of Protocol was created. So, statement 2 is also appointment to the Supreme Court) for correct. appointment by the President of India, as his successor.

www.iasgatewayy.com

UPSC NOVEMBER- 2019 DAILY CURRENT AFFAIRS CLASS TEST - 01 5. Consider the following statements Explanation: regarding International Solar Alliance Agricultural Produce Market Committee Acts of (ISA): respective states are responsible for markets in 1. ISA was launched during COP 21 of agricultural products. UNFCCC in 2015 which took place at Paris, France. 7. Which one of the following countries of 2. It is a de-jure treaty based International South-West Asia does not open out to the Intergovernmental Organization, Mediterranean Sea? headquartered at New York. (a) Syria Choose the correct answer using the code given (b) Jordan below: (c) Lebanon (a) 1 only (d) lsrael (b) 2 only Answer: b) (c) Both 1 and 2 Explanation: (d) Neither 1 nor 2 Jordan does not open out to the Mediterranean Answer: a) Sea. Explanation: The International Solar Alliance (ISA) was 8. ‘IBA100’ recently seen in news is referred launched on 30th November 2015 in Paris as an to inter-governmental treaty organisation by India (a) Newly discovered element and France. (During COP 21). So, statement 1 is (b) Indian Brain Atlas correct. (c) Missile jointly developed by and It is a de-jure treaty based International Brazil Intergovernmental Organization, headquartered (d) Vaccine for Dengue at Gurugram, India. So, statement 2 is incorrect. Answer: b) Explanation: 6. In India, markets in agricultural products IIIT-Hyderabad has created the first Indian brain are regulated under the atlas named IBA100. (a) Essential Commodities Act, 1955 The Indian brain atlas reveals that Indian brain (b) Agricultural Produce Market Committee is smaller in height, width, and volume compared Act enacted by States to the western and eastern populations. (c) Agricultural Produce (Grading and This study will help in the early diagnosis of Marking) Act, 1937 brain diseases like Alzheimer’s. (d) Food Products Order, 1956 and Meat and Food Products Order, 1973 Answer: b)

www.iasgatewayy.com

UPSC NOVEMBER- 2019 DAILY CURRENT AFFAIRS CLASS TEST - 01 9. Which of the following cities are included in (d) All of the above the UNESCO Creative Cities Network Answer: d) (UCCN)? Explanation: 1. Varanasi The GHI ranks countries on a 100-point 2. Jaipur scale, with 0 being the best score (no hunger) 3. Madurai and 100 being the worst. Values less than 10 4. Mumbai reflect low hunger, values from 20 to 34.9 indicate serious hunger; values from 35 to 49.9 5. Hyderabad are alarming; and values of 50 or more are Select the correct answer using the codes given extremely alarming. below: The report is a peer-reviewed publication (a) 1, 2, 3 and 4 only released annually by Welthungerhilfe and (b) 1, 3, 4 and 5 only Concern Worldwide. (c) 1, 2, 4 and 5 only The GHI scores are based on a formula that (d) All the above captures three dimensions of hunger— Answer: c) insufficient caloric intake, child under nutrition, Explanation: and child mortality—using four component Recently Hyderabad and Mumbai were included indicators. under the categories of Gastronomy and Film. India is ranked 102 of 117 countries in the Global Other cities included in the UCCN are Varanasi Hunger Index 2019, behind its neighbours Nepal, (Music), Jaipur (Crafts and Folk Art) and Pakistan and Bangladesh. Chennai (Music).

10. Consider the following statements with

regard to Global Hunger Index:

1. High Income countries are not taken for

ranking under the index.

2. India was ranked serious category in

this index.

3. India was well behind Pakistan, Nepal

and Bangladesh.

Which of the following above statements is/are correct? (a) 1 only (b) 1 and 3 only (c) 1 and 2 only

www.iasgatewayy.com

UPSC NOVEMBER- 2019 DAILY CURRENT AFFAIRS CLASS TEST - 02 1. Tapan Ray Committee recently seen in the Explanation: news recently was constituted for which of Only The President can appoint a judge of a high the following purposes? court as an acting chief justice of the high court. (a) To suggest for revamping of PM National The Supreme Court has recently ruled that a Disaster Relief Fund judge who retired as an Acting Chief Justice of a (b) To study issues in the Prevention of High Court cannot claim the pension of a regular Money Laundering Act Chief Justice (CJ). (c) To review the framework for Core SC held that an Acting CJ needs to be given the Investment Companies pension of a CJ only for the period for which he (d) To bring reforms in the banking sector served as a CJ and not more. Answer: c) Only for the limited purpose of salary, such an Explanation: Acting Chief Justice is treated on a par with the The Reserve Bank of India (RBI) has set up a Chief Justice and not for any other purpose, working group to review the regulatory and more particularly pension. supervisory framework for core investment companies (CIC). 3. Recently which of the following country The group will be headed by Tapan Ray, non- released the special commemorative stamp executive chairman, Central Bank of India and on ‘Ramayana’? former Secretary in the Ministry of Corporate (a) Malaysia Affairs. (b) Sri Lanka 2. Consider the following statements (c) Indonesia regarding Chief Justice of High Court: (d) Thailand

1. Chief Justice of India (CJI) can appoint Answer: c) a judge of a high court as an acting chief Explanation: justice of the high court. Indonesia has released a special commemorative 2. A judge who retired as an Acting Chief stamp on the theme of Ramayana to mark the Justice of a High Court can claim the 70th anniversary of establishment of its pension of a regular Chief Justice (CJ). diplomatic ties with India. Which of the statement(s) given above is/are A specially signed version of the stamp will be on incorrect? display at the Philately Museum in Jakarta. (a) 1 only 4. ‘RCEP’ includes which of the following (b) 2 only countries? (c) Both 1 and 2 (d) Neither 1 nor 2 1. Australia Answer: c) 2. New Zealand 3. India www.iasgatewayy.com

UPSC NOVEMBER- 2019 DAILY CURRENT AFFAIRS CLASS TEST - 02

4. US (c) 1 and 3 only (d) 1, 2 and 3 5. Australia 6. China Answer: d) Select the correct answer from the codes given Explanation: below: All the statements are correct. (a) 1, 2, 3 and 4 only (b) 1, 2, 3, 4 and 6 only 6. Consider the following statements (c) 1, 2, 3, 5 and 6 only regarding electoral bonds in India: (d) All the above 1. Only the political parties that has secured at least one per cent of the votes Answer: c) polled in the most recent election are Explanation: eligible to receive these bonds. RCEP is proposed between the ten member 2. Private companies are not eligible to states of the Association of Southeast Asian donate electoral bonds. Nations (ASEAN) (Brunei, Burma [Myanmar], Choose the correct statement using the code Cambodia, Indonesia, Laos, Malaysia, the given below: Philippines, Singapore, Thailand, Vietnam) and (a) 1 only the six states with which ASEAN has existing (b) 2 only FTAs (Australia, China, India, Japan, South (c) Both 1 and 2 Korea and New Zealand). (d) Neither 1 nor 2

5. Consider the following statements Answer: a) regarding the ‘Electoral Bonds’: Explanation: 1. Electoral bonds will allow donors to pay Every party that is registered under section 29A political parties using banks as an of the Representation of the Peoples Act, 1951 intermediary. (43 of 1951) and has secured at least one per cent 2. Electoral bonds may be purchased by a of the votes polled in the most recent Lok Sabha citizen of India, or entities incorporated or State election has been allotted a verified or established in India. account by the Election Commission of India. 3. A person being an individual can buy Electoral bond transactions can be made only via electoral bonds, either singly or jointly this account. So, statement 1 is correct. Private companies are also eligible to donate to with other individuals. political parties through electoral bonds. So, Which of the statement(s) given above is/are statement 2 is incorrect. correct?

(a) 1 only (b) 1 and 2 only

www.iasgatewayy.com

UPSC NOVEMBER- 2019 DAILY CURRENT AFFAIRS CLASS TEST - 02 7. Consider the following statements Explanation: regarding National Crime Record Bureau Vijayanagara School: More than one mandaps (NCRB): were built in each temple. The central mandap 1. It is a statutory body created under the came to be known as Kalyan Mandap. provisions of RTI Act, 2005. 2. It functions under the Ministry of 9. Right to Privacy is protected as an intrinsic Personnel, Public Grievances and part of Right to Life and Personal Liberty. Pensions. Which of the following in the Constitution Choose the correct statement using the code of India correctly and appropriately imply given below: the above statement? (a) 1 only (a) Article 14 and the provisions under the (b) 2 only 42nd Amendment to the Constitution. (c) Both 1 and 2 (b) Article 17 and the Directive Principles of (d) Neither 1 nor 2 State Policy in Part IV (c) Article 21 and the freedoms guaranteed in Answer: d) Part III Explanation: (d) Article 24 and the provisions under the It has been created by a government resolution 44th Amendment to the Constitution. based on the recommendations of the National Police Commission (1977-1981) and the MHA’s Answer: c) Task force (1985). So, statement 1 is incorrect. Explanation: NCRB, headquartered in New Delhi, was set-up Article 21 guarantees the Right to life and in 1986 under the Ministry of Home Affairs to personal liberty under Part III of the function as a repository of information on crime Constitution. The historic fallout of the nine- and criminals so as to assist the investigators in judge Bench judgment, declaring privacy as linking crime to the perpetrators. So, Statement2 intrinsic to life and liberty and an inherent right is incorrect. protected by Part III of the Constitution, is that an ordinary man can now directly approach the 8. Building ‘Kalyaana Mandapas’ was a Supreme Court and the High Courts for violation notable feature in the temple construction of his fundamental right under the Constitution. in the kingdom of By making privacy an intrinsic part of life and (a) Chalukya liberty under Article 21, it is not just a citizen, but (b) Chandela anyone, whether an Indian national or not, can (c) Rashtrakuta move the constitutional courts of the land under (d) Vijayanagara Articles 32 and 226, respectively, to get justice.

Answer: d)

www.iasgatewayy.com

UPSC NOVEMBER- 2019 DAILY CURRENT AFFAIRS CLASS TEST - 02 10. Which one of the following is the national aquatic animal of India? (a) River Dolphin (b) Olive ridley turtle (c) Gangetic Dolphin (d) Gharial

Answer: c) Explanation: The Gangetic dolphins have been declared as the National Aquatic Animal of India. River Dolphin is the National Aquatic Animal of India. The Ministry of Environment and Forests notified the Ganges River Dolphin as the National Aquatic Animal on 18th May 2010. This mammal is also said to represent the purity of the holy Ganga as it can only survive in pure and fresh water.

www.iasgatewayy.com

UPSC NOVEMBER- 2019 DAILY CURRENT AFFAIRS CLASS TEST - 04 1. The term "lonewolf attack" recently in news 3. Consider the following Statements about refers to Indian Border (a) A kind of gene editing technology 1. India has the largest land border with (b) Melting of largest glacier in Arctic region Bangladesh (c) Individuals undertaking violent acts of 2. Indo-Bangladesh border is guarded by terrorism outside a command structure. Assam Rifles (d) None of the above Which of the above statement(s) is/are Answer: c) correct? Explanation: The term lone wolf is used to (a) 1 only distinguish terrorist actions carried out by (b) 2 only individuals from those coordinated by large (c) Both 1 and 2 groups (d) Neither 1 nor 2

Answer: a) 2. Consider the following statements Explanation: Bangladesh has a 4,096 regarding Dhanush, which is recently in kilometres border with India. It is guarded by news: the Border Security Force

1. It is the first indigenous artillery gun produced in India. 4. "SILAM" is a model for explaining 2. It is indigenously upgraded variant of (a) Ozone hole Size the Swedish-Bofors gun. (b) Chemical Pollutant Dispersal Choose the correct answer using the code (c) Unemployment to inflation ratio trend given below: (d) Industrial Solid Waste treatment (a) 1 only Answer: b) (b) 2 only Explanation: SILAM is a global-to-meso- (c) Both 1 and 2 scale dispersion model developed for (d) Neither 1 nor 2 atmospheric composition, air quality, and Answer: c) emergency decision support applications, as Explanation: well as for inverse dispersion problem Dhanush is the first indigenous artillery gun solution.

with a calibre of 155mm x 45mm. It is the 5. Consider the following statements first long range artillery gun to be produced regarding IRNSS: in India, having a range of 38 km. So, 1. IRNSS system consists of 4 statement 1 is correct. Geosynchronous satellites and 3 Dhanush is the indigenously upgraded geostationary satellites. variant of the Swedish-Bofors gun imported 2. IRNSS-1I was the last satellite that was in the 1980s. So, statement 2 is also correct. launched in the IRNSS series.

www.iasgatewayy.com

UPSC NOVEMBER- 2019 DAILY CURRENT AFFAIRS CLASS TEST - 04 Choose the correct answer using the code (a) 1 and 2 only given below: (b) 2 and 3 only (a) 1 only (c) 1 and 3 only (b) 2 only (d) 1, 2 and 3 (c) Both 1 and 2 Answer: d) (d) Neither 1 nor 2 Explanation: Answer: c) A High-Level Advisory Group on Trade Policy Explanation: (HLAG) headed by Surjit S Bhalla has It is a constellation of total 7 satellite recently suggested the government to issue launched in space and a ground facility on ‘Elephant Bonds’ which would recover up to land to receive signals from space satellites. 3 $500 billion of black money that has been of its satellite Located in Geostationary orbit stashed overseas. and 4 are inclined to geosynchronous orbit. An Elephant Bond is a 25-year sovereign However full NAVIC system has 9 satellite, 2 bond (a bond issued by a national on ground in standby mode. So, the government). statement 1 is correct. This bond is issued to those people who IRNSS-1I was launched last year to replace declare their previously undisclosed income India’s first navigation satellite IRNSS-1A, and are then bound to invest 50% of that whose three Rubidium atomic clocks had amount in these securities. stopped working. It was also the last satellite The fund gathered by the issuance of these to be launched in the IRNSS series. So, bonds is utilized to finance infrastructure statement 2 is also correct. projects only which was recommended by the HLAG. 6. Consider the following statements One of the key features of the proposed regarding the Elephant Bonds: mechanism is that those disclosing their 1. Elephant Bonds are issued under the black money will receive immunity from all recommendation of Surjit S Bhalla local laws including those under foreign Committee. exchange, black money laws, and taxation 2. This bond is issued to those people who laws. declare their previously undisclosed 7. ‘Bill and Keep’ sometimes seen in news income. refer to 3. The fund gathered by the issuance of (a) It is the charge payable between the these bonds is utilized to finance network providers. infrastructure projects only. (b) Agreement between the network Which of the statement(s) given below are providers to terminate calls from one to correct? other network at no charge.

www.iasgatewayy.com

UPSC NOVEMBER- 2019 DAILY CURRENT AFFAIRS CLASS TEST - 04 (c) It is the agreement between the telecom (a) 1 only sector and GOI to manage the customer (b) 1 and 2 only services. (c) 3 only (d) None of the above (d) 1 and 3 only Answer: b) Answer: c) Explanation: Explanation: According to the OECD, Bill and Keep is The Sea Surface Temperatures (SST) was defined as "A pricing scheme for the two-way higher than normal over the Arabian Sea interconnection of two networks under which since October, towards the culmination of the the reciprocal call termination charge is zero - southwest monsoon season. that is, each network agrees to terminate calls Generally, such higher SSTs favour the from the other network at no charge". formation of ocean systems like depression Bill and keep represents a modern approach that can, over a short period of time, intensify to interconnection charging in which the into cyclones or super cyclones while at sea. networks recover their costs only from their The occurrence of cyclones is more frequent own customers rather than from the sending and common in the Bay of Bengal compared network. to the Arabian Sea. However, in recent years, the latter has also 8. Which one of the following statements seen its share of cyclones. In 2019, four is/are not correct about Cyclones? cyclones have formed over the Arabian Sea. 1. The Sea Surface Temperatures (SST) This growing cyclogenesis and formation of was higher than normal over the intense cyclones in the Arabian Sea could be Arabian Sea since October, towards the linked to global warming and is seen as an culmination of the southwest monsoon effect of climate change.

season. 9. ‘Glyphosate’, recently seen in news is

2. Higher SSTs favour the formation of referred to ocean systems like depression that can, (a) A Compound present in herbicides. over a short period of time, intensify (b) Newly found dwarf planet into cyclones or super cyclones while at (c) Medicine for arthritis sea. (d) A Gas which is found in mesosphere 3. The occurrence of cyclones is more Answer: a) frequent and common in the Arabian Explanation: Sea compared to the Bay of Bengal. German pharma company Bayer is facing Select the correct answer using the codes thousands of lawsuits over its herbicides for given below: their alleged cancer risks.

www.iasgatewayy.com

UPSC NOVEMBER- 2019 DAILY CURRENT AFFAIRS CLASS TEST - 04 The herbicides are based on a compound in the environment health policy and deaths called glyphosate. First developed in 1970, due to air pollution categories. glyphosate is scientifically N- The 10th EPI report ranks 180 countries on 24 (phosphonomethyl) glycine under the IUPAC performance indicators across 10 categories system of nomenclature. covering environmental health and ecosystem It is applied to the leaves of plants to kill vitality. The EPI said air quality remains the weeds. It is widely used in India, too. leading environmental threat to public health This weed killer in India goes by various brand names, including Roundup, Glycel, and Brake. In 2015, the World Health Organization’s International Agency for Research on Cancer (IARC) published a study that found glyphosate is “probably carcinogenic to humans”

10. ‘Environmental Performance Index’was released by (a) Minstry of Environment, Forest and Climate Change (b) Greenpeace International (c) United Nations Environment Programme (d) None of the Above

Answer: d) Explanation: India is among the bottom five countries on the Environmental Performance Index 2018, plummeting 36 points from 141 in 2016, according to a biennial report by Yale and Columbia Universities along with the World Economic Forum. While India is at the bottom of the list in the environmental health category, it ranks 178 out of 180 as far as air quality is concerned Its overall low ranking — 177 among 180 countries — was linked to poor performance

www.iasgatewayy.com

UPSC NOVEMBER- 2019 DAILY CURRENT AFFAIRS CLASS TEST - 05

1. The term ICEDASH has been recently in So, A, B and D are removed. Thus, by News. It refers to elimination, we are left with Answer C. (a) Cryogenic technology deployed by ISRO (b) Arctic Ice cutting ship by Russia 3. Consider the following about Shanghai (c) Ease of Doing Business monitoring Cooperation Organization dashboard of the Indian Customs 1. While India has been given membership (d) Broken piece of iceberg from Antarctic of SCO, Pakistan remains a Dialogue Answer: c) partner Explanation: ICEDASH is an Ease of Doing 2. The Council of Heads of State is the top Business monitoring dashboard of the Indian decision-making body in the SCO Customs helping the public see the daily Which of the above statement(s) is/are Customs clearance times of import cargo at Incorrect? various ports and airports. (a) 1 only

(b) 2 only 2. Which of the following countries are the (c) Both 1 and 2 'free-trade partners' of ASEAN? (d) Neither 1 nor 2 1. Australia Answer: a) 2. Canada Explanation: Both countries are members 3. China of SCO

4. India 5. Japan 4. “Adolescents, Diets and Nutrition: Growing 6. USA Well in a Changing World” report was Choose the correct answer using code given released by which of the following below: organisation / Institution? (a) 1, 2, 4 and 5 (a) Medicos Sans Frontiers (b) 3, 4, 5 and 6 (b) UNICEF (c) 1, 3, 4 and 5 (c) UNESCO (d) 2, 3, 4 and 6 (d) IMF Answer: c) Answer: b) Explanation: RCEP about ASEAN plus its Explanation: As per the report free trade partners which includes: China, “Adolescents, Diets and Nutrition: Growing India, Japan, Korea, Australia and New Well in a Changing World 2019” released by Zealand. UNICEF more than 80 % of adolescents in Canada (#2) and USA (#6) is definitely not India suffer from “hidden hunger” (a form of there. undernutrition), and less than 10 % of boys and girls eat fruits and eggs daily.

www.iasgatewayy.com

UPSC NOVEMBER- 2019 DAILY CURRENT AFFAIRS CLASS TEST - 05 5. “Global Microscope: The enabling (b) Chaitya is a place of worship, while Vihara environment for Financial Inclusion” report is the dwelling place of the monks is published by (c) Chaitya is the stupa at the far end of the (a) The Economist Intelligence Unit cave, while Vihara is the hall axial to it (b) Financial Action Task Force (d) There is no material difference between (c) World Bank the two (d) International Monetary Fund Answer: b) Answer: a) Explanation: Chaityas refer to the halls Explanation: The Economist Intelligence enclosing the stupas. Chaityas were probably Unit has released the 2019 edition of “Global constructed to hold large numbers of Microscope: The enabling environment for devotees for prayer. Viharas on the other Financial Inclusion report”. hand are constructions built in ancient India As per the report, India was placed at the 5th in order to provide resting places for the spot in the nations having the most conducive wandering Buddhist monks. environments for inclusive finance. 8. Consider the following statements with 6. Consider the following statements with regard to Environment Pollution Control respect to Cyclone Maha: Board 1. Cyclone was developed in Bay of Bengal 1. It is mainly created to protect and 2. The name was given by Kuwait control environment pollution in Which of the above statement(s) is/are correct? National Capital Region. (a) 1 only 2. It enforces Graded Response Action (b) 2 only Plan in all states of India as per the (c) Both 1 and 2 pollution level. (d) Neither 1 nor 2 Which of the above give statement(s) is/are Answer: d) correct? Explanation: Cyclone was developed in (a) 1 only Arabian Sea. The name was given by Oman (b) 2 only (c) Both 1 and 2 only 7. Some Buddhist rock-cut caves are called (d) Neither 1 nor 2 Chaityas, while the others are called Answer: a) Viharas. What is the difference between the Explanation: EPCA was constituted with two? the objective of ‘protecting and improving’ (a) Vihara is a place of worship, while Chaitya the quality of the environment and is the dwelling place of the monks ‘controlling environmental pollution’ in the National Capital Region. To enforce Graded

www.iasgatewayy.com

UPSC NOVEMBER- 2019 DAILY CURRENT AFFAIRS CLASS TEST - 05 Response Action Plan (GRAP) in NCR as per water body. All the water entering Danakil the pollution levels. evaporates, and no streams flow out from its extreme environment. It is covered with more 9. Which of the following countries involve in than 10 lakh tonnes of salt. joint Exercise 'Dustlik 2019' (a) India and United States (b) India and Maldives (c) India and Sri Lanka (d) India and Uzbekistan Answer: d) Explanation: Dustlik 2019 will be conducted for about ten days in November 2019 at Tashkent. This is a joint exercise of the armies of India and Uzbekistan. The exercise will enable sharing of best practices and experiences between the Armed Forces of the two countries and would lead to greater operational effectiveness. The primary focus will be on counterterrorism.

10. The "Danakil Depression" which is considered hostile even for Extremophile microbes is in (a) Sudan (b) Kenya (c) Ethiopia (d) Somalia Answer: c) Explanation: The Danakil Depression in north eastern Ethiopia is one of the world’s hottest places, as well as one of its lowest, at 100 metres below sea level. At the northern end of the Great Rift Valley, and separated by live volcanoes from the Red Sea, the plain was formed by the evaporation of an inland

www.iasgatewayy.com

UPSC NOVEMBER- 2019 DAILY CURRENT AFFAIRS CLASS TEST - 06 1. Consider the following statements Explanation: regarding “Skills Build platform”: India and Russia are finalising a defence 1. It is launched by Ministry of Skill agreement named “Agreement on Reciprocal Development & Entrepreneurship in Logistics Support (ARLS)” that will simplify collaboration with Wipro. interoperability and enable military 2. This digital platform was launched to platforms to receive support and supplies provide personal assessment of the across bases in both nations.

cognitive capabilities and personality of 3. Consider the following statements the students. regarding the NASA’s Voyager 2: Choose the correct answer using the code given below: 1. Voyager 2 is the only probe ever to study (a) 1 only Neptune and Uranus during planetary (b) 2 only flybys. (c) Both 1 and 2 2. It is the only spacecraft to have visited (d) Neither 1 nor 2 all four gas giant planets — Jupiter, Answer: (b) Saturn, Uranus and Neptune. Explanation: 3. Voyager spacecraft has confirmed that “Skills Build platform” was launched by the plasma inside the heliosphere is Ministry of Skill Development & denser than the plasma in local Entrepreneurship in Collaboration with IBM. interstellar space. So, Statement 1 is incorrect. Which of the statement(s) given above is/are This digital platform will provide a personal correct? assessment of the cognitive capabilities and (a) 1 only personality of the students. So, Statement 2 is (b) 1 and 2 only correct. (c) 1 and 3 only (d) 1, 2 and 3 2. Agreement on Reciprocal Logistics Support Answer: (b) (ARLS), recently in news is signed between Explanation: India and Voyager spacecraft has confirmed that the (a) Russia plasma inside the heliosphere is significantly (b) U.S.A less dense and less cold than the plasma in (c) Japan local interstellar space. (d) Vietnam Answer: (a)

www.iasgatewayy.com

UPSC NOVEMBER- 2019 DAILY CURRENT AFFAIRS CLASS TEST - 06 4. Consider the following statements 5. Consider the following statements: regarding the National Health Profile 1. Dwarf Planets are celestial objects (NHP)-2019 orbiting the Sun that are large enough 1. Socio-economic indicators and for their gravity to pull them into a demographic indicators are part of the spherical shape. profile. 2. Names of celestial bodies are finally 2. Acute Respiratory Infections (ARI) approved by a committee at the accounted for majority of morbidity last International Astronomical Union year in the non-communicable disease (IAU). category. Which of the statement(s) given above is/are Which of the statement(s) given above is/are correct? incorrect? (a) 1 only (a) 1 only (b) 2 only (b) 2 only (c) Both 1 and 2 (c) Both 1 and 2 (d) Neither 1 nor 2 (d) Neither 1 nor 2 Answer: b) Answer: d) Explanation: Explanation: Minor planets or dwarf planets are celestial The NHP highlights substantial health objects orbiting the Sun that are not large information under major indicators viz. enough for their gravity to pull them into a demographic indicators (population and vital spherical shape. statistics), socio-economic indicators Names of celestial bodies are finally approved (education, employment, housing and by a committee at the International amenities, drinking water and sanitation) and Astronomical Union (IAU), a global health status indicators (incidence and organisation of professional astronomers, prevalence of common communicable and which also decides on definitions of non-communicable diseases and RCH), etc. fundamental astronomical and physical It was released for the first time in 2005. constants. 2019 is the 14th year the NHP is being In the case of small Solar System bodies, the released. A digital version of the report was discoverer has the privilege to suggest the also released. name. Acute Respiratory Infections (ARI) accounted The discoverer holds this privilege for 10 for 69.47% (majority) of morbidity last year years since the discovery. which was the highest in the communicable But there is a process to be followed, and not disease category. all names are acceptable.

www.iasgatewayy.com

UPSC NOVEMBER- 2019 DAILY CURRENT AFFAIRS CLASS TEST - 06 6. The Drukpa Lineage, recently in news is 8. According to the recently released Waste (a) Matriarchal Lineage in Arunachal Land Atlas, Pradesh 1. The changes in wastelands between (b) A thousand-year-old Buddhist tradition 2008-09 and 2015-16 have been that began in the Himalayas. presented in the Atlas. (c) A lineage in South India which has 2. A reduction in wasteland area was hereditary disease, whose DNA has been observed in the category of land with decoded dense scrub (d) None of the above 3. The wastelands have undergone Answer: b) negative change in the state of Explanation: Rajasthan Union Minister of State for Tourism and Which of the above statements are correct? Culture met the Kung Fu Nuns of the Drukpa (a) 1 and 2 only Order. (b) 2 and 3 only The Kung Fu Nuns recently received the Asia (c) 1 and 3 only Society’s prestigious Game Changer Award in (d) All the above New York for their path-breaking work to Answer: a) empower women and dismantle gender Explanation: stereotypes in the Himalayas. The wastelands have undergone positive

change in the states of Rajasthan (0.48 Mha), 7. Global Tuberculosis Report 2019 was Bihar (0.11 Mha), Uttar Pradesh (0.10 Mha), recently published by which of the Andhra Pradesh (0.08 Mha), Mizoram (0.057 following? Mha), Madhya Pradesh (0.039 Mha), Jammu (a) Food and Agricultural Organisation & Kashmir (0.038 Mha) and West Bengal (b) World Health Organisation (0.032 Mha). (c) Doctors without borders

(d) Codex Alimentarius commission 9. Recently unvield , the PANNAI app is used Answer: b) for Explanation: (a) Pest-disease Advance Notification According to the Global Tuberculosis Report (b) PAN India LPG connection Mobility 2019 released by the World Health (c) Particulate and Non-Particulate matter Organization (WHO), the world is not on pollution monitoring track to reach the 2020 milestones of the (d) Remote Sensing ‘End TB Strategy’. Answer: a)

www.iasgatewayy.com

UPSC NOVEMBER- 2019 DAILY CURRENT AFFAIRS CLASS TEST - 06 Explanation: Pannai, a new mobile app for pest-disease warning, developed by the M.S. Swaminathan Research Foundation (MSSRF).Pannai, a new mobile app for pest-disease warning, developed by the M.S. Swaminathan Research Foundation (MSSRF).

10. Consider the following statements: 1. The minimum age prescribed for any person to be a member of Panchayat is 25 years. 2. A Panchayat reconstituted after premature dissolution continues only for the remainder period. Which of the statements given above is/are correct? (a) 1 only (b) 2 only (c) Both 1 and 2 (d) Neither 1 nor 2 Answer: (b) Explanation: The minimum age for contesting elections to panchayats is 21 years. In case of dissolution of Panchayats before expiry of its term, it is mandatory to hold elections within 6 months of its dissolution. Every Panchayat shall continue for full term of 5 years. However, a Panchayat reconstituted after premature dissolution shall continue for only the reminder of the period. If the period is less than 6 months then it is not mandatory to held the elections.

www.iasgatewayy.com

UPSC NOVEMBER- 2019 DAILY CURRENT AFFAIRS CLASS TEST - 07 1. Consider the following about Polluter Pays through additional forest and tree cover Principle by 2030. 1. It states that the producer of pollution Which of the above statements are correct? should bear the costs of managing it. (a) 1 only 2. It is part of the Agenda 21 (b) 1 and 3 only Which of the above statement(s) is/are (c) 2 and 3 only incorrect? (d) All the above (a) 1 only Answer: d) (b) 2 only Explanation: All the above are factually (c) Both 1 and 2 correct statements about India's NDC (d) Neither 1 nor 2 commitments Answer: b) Explanation: Its part of Rio Declaration of 3. Consider the following statements 1992. regarding Alzheimer’s disease: Principle 16 states that " National authorities 1. It is a chronic disease which usually should endeavour to promote the affects the lungs. internalization of environmental costs and 2. It is usually spread by Culex the use of economic instruments, taking into mosquitoes. account the approach that the polluter Choose the correct answer using the code should, in principle, bear the cost of given below: pollution, with due regard to the public (a) 1 only interest and without distorting international (b) 2 only trade and investment. (c) Both 1 and 2

(d) Neither 1 nor 2 2. Consider the following about the targets of Answer: d) India’s Nationally Determined Explanation: Contributions (NDCs) It is a progressive brain disorder that 1. To reduce the emissions intensity of its typically affects people older than 65. When it GDP by 33 to 35 per cent by 2030 from affects younger individuals, it is considered 2005 level. early onset. So, Statement 1 is incorrect. 2. To achieve about 40 per cent cumulative There is no cure for Alzheimer’s, because its electric power installed capacity from exact causes are not known. There is a degree non-fossil fuel-based energy resources of consensus in the scientific community that by 2030 Alzheimer’s involves two proteins, called beta 3. To create an additional carbon sink of amyloids and tau. When levels of either 2.5 to 3 billion tonnes of CO2 equivalent protein reach abnormal levels in the brain, it

www.iasgatewayy.com

UPSC NOVEMBER- 2019 DAILY CURRENT AFFAIRS CLASS TEST - 07 leads to the formation of plaque, which gets 5. Consider the following statements deposited between neurons, damaging and regarding Paris Agreement: disrupting nerve cells. So, Statement 2 is also 1. This agreement doesn’t state any incorrect. standard procedure for any country leaving the pact.

4. Consider the following statements 2. U.S.A cannot join the pact again, once it regarding National Green Tribunal (NGT): leaves the Agreement. 1. It is created under the provisions of Choose the correct answer using the code Environmental protection Act,1986. given below: 2. NGT is supposed to follow the (a) 1 only procedure laid down under the Code of (b) 2 only Civil Procedure 1908. (c) Both 1 and 2 3. Majority of the cases dealt by NGT falls (d) Neither 1 nor 2 under the ambit of Wildlife (Protection) Answer: d) Act, 1972. Explanation: Choose the correct answer using the code A country can leave the pact through Article given below: 28 of the Paris Agreement which allows (a) 1 only (b) 1 and 2 only countries to leave the Paris Agreement. It also (c) 2 and 3 only (d) None of the above lays down the process for leaving the Answer: d) agreement. So, Statement 1 is incorrect. Explanation: It can re-join the agreement whenever it It is a specialized body set up under the wishes to do so. There is no bar on a country National Green Tribunal Act (2010) for re-joining the Paris Agreement. So, statement effective and expeditious disposal of cases. 2 is also incorrect.

So, statement 1 is incorrect. 6. With respect to Kisan Credit Card Scheme, The Tribunal is not bound by the procedure consider the following laid down under the Code of Civil Procedure 1. It was launched to provide formal credit 1908, but shall be guided by principles of support to the farmers. 'natural justice'. So, Statement 2 is also 2. Commercial Banks are exempted from incorrect. implementation of the scheme. Wildlife (Protection) Act, 1972 and Scheduled 3. Animal Husbandry farmers are eligible Tribes and Other Traditional Forest Dwellers to get credit under the scheme. (Recognition of Forest Rights) Act, 2006 have Choose the correct answer using the code been kept out of NGT’s jurisdiction. So, given below: statement 3 is also incorrect.

www.iasgatewayy.com

UPSC NOVEMBER- 2019 DAILY CURRENT AFFAIRS CLASS TEST - 07 (a) 1 only Explanation: Kyasanur forest disease (b) 1 and 2 only (KFD) is a tick-borne viral haemorrhagic (c) 2 and 3 only fever endemic to South India (d) 1 and 3 only Answer: d) 9. Recently, ‘oilzapper’ was in the news. What Explanation: is it? The Kisan Credit Card Scheme is (a) It is an eco-friendly technology for the implemented by Commercial Banks, RRBs, remediation of oil sludge and oil spills. Small Finance Banks and Cooperatives. (b) It is the latest technology developed for Hence statement 2 is not correct. under- sea oil exploration.

(c) It is a genetically engineered high biofuel 7. Consider the ollowing statements about yielding maize variety. National Integration Council (d) It is the latest technology to control the 1. It is a constitutional body established accidentally caused flames from oil wells. under article 243. Answer: a) 2. It is chaired by President of India. Explanation: Oilzapper is an eco-friendly 3. Its Council Members includes technology for the remediation of oil sludge representatives of industry, business and oil spills.

and trade unions. 10. Joint Comprehensive Plan of Action is an Choose the correct answer using the code given below: agreement on the Iranian nuclear program (a) 1 only involving Iran and (b) 1 and 2 only 1. European Union (c) 3 only 2. China (d) 2 and 3 only 3. United Kingdom Answer: c) 4. Germany Explanation: It is an extra-constitutional 5. France body, chaired by: Prime Minister of India. 6. Russia (a) 1,2,4,5 and 6 only 8. Kyasanur Forest disease is endemic to (b) 2,3,4,5 and 6 only (a) North-East India (c) 3,4,5 and 6 only (b) Foot Hills of Himalayas (d) 1,2,3,4,5 and 6 (c) South India Answer: d) (d) Gir Forest surroundings of Gujarat

Answer: c)

www.iasgatewayy.com

UPSC NOVEMBER- 2019 DAILY CURRENT AFFAIRS CLASS TEST - 07 Explanation: It is an agreement on the Iranian nuclear program reached in Vienna on July 14, 2015, between Iran and the P5+1 (the five permanent members of the United Nations Security Council—China, France, Russia, United Kingdom, United States—plus Germany) together with the European Union.US has currently withdrawn from it.

www.iasgatewayy.com

UPSC NOVEMBER- 2019 DAILY CURRENT AFFAIRS CLASS TEST - 08 1. Consider the following statements 2. Consider the following statements regarding the Central Armed Police Force regarding (CAPF): 1. India’s contribution to world’s milk 1. The CAPF comes under the production is about 20% as in 2018. administrative control of Ministry of 2. India likely to be milk surplus country defence. by next 15 years. 2. It includes the Indian Army, Indian 3. If India signed RCEP deal, then it would Navy and Indian Air force and the be a big blow for Dairy farmers. Indian Coast Guard. Choose the correct answer using the code Which of the statement(s) given above is/are given below: correct? (a) 1, 2 only (a) 1 only (b) 2 ,3 only (b) 2 only (c) 1 and 3 only (c) Both 1 and 2 (d) All of the above (d) Neither 1 nor 2 Answer: d) Answer: d) Explanation: Explanation: All the statements are true regarding as per The CAPF comes under the administrative NITI Aayog Report. control of Ministry of Home Affairs. According to NITI Aayog report, India will be milk surplus country in 2033. It includes the Indian Army, Indian Navy and 3. Choose the incorrect statement regarding Indian Air force and the Indian Coast Guard. East Asia Summit: They are the, Border Security Force (BSF), (a) It’s a regional forum of 16 countries that Central Reserve Police Force (CRPF), Central meets annually. Industrial Security Force (CISF), Indo- (b) India is the only member among SAARC Tibetan Border Police (ITBP), Assam Rifles countries to be a member of EAS. (AR), National Security Guard (NSG) and (c) It is attended by all the member countries Sashastra Seema Bal (SSB). of ASEAN, QUAD and Mekong Ganga

Cooperation. At present, the Assam Rifles, a Central (d) No Country from Gulf cooperation paramilitary force, is under the Council (GCC) and European Union is its administrative control of the MHA and under member. the operational control of the Army, i.e. the Answer: a) Ministry of Defence. Explanation: East Asia Summit consists of

18 countries. ((ASEAN+6) + Russia + USA)

www.iasgatewayy.com

UPSC NOVEMBER- 2019 DAILY CURRENT AFFAIRS CLASS TEST - 08 4. In the context of India Justice Report, 2019, Choose the correct answer using the code consider the following statements: given below: 1. It was released by the National Crime (a) 1 only Records Bureau. (b) 2 only 2. The entities are based on the four pillars (c) Both 1 and 2 of justice delivery such as police, (d) Neither 1 nor 2 judiciary, prisons and legal aid. Answer: a) Explanation: 3. The report has stated that Maharashtra It is not empowered to intervene in the affairs has topped the report in the large- of the country. Recently ICAO said they can’t medium states category. intervene in Pakistan denial of airspace to Which of the statement(s) given above is/are India. incorrect?

(a) 1 only 6. The ‘Financial stability Report’ is released (b) 1 and 2 only by (c) 2 and 3 only (a) Reserve Bank of India (d) 1, 2 and 3 (b) Financial Stability Development Council Answer: a) (c) SEBI Explanation: (d) NABARD The India Justice Report 2019, commissioned Answer: a) by Tata Trusts, was prepared by groups like Explanation: Centre for Social Justice, Common Cause, The Reserve Bank of India recently releases Commonwealth Human Rights Initiative, the Financial Stability Report (FSR). DAKSH, Tata Institute of Social Sciences - The FSR reflects the collective assessment of Prayas and Vidhi Centre for Legal Policy.

the Sub-Committee of the Financial Stability

and Development Council (FSDC) on risks to 5. Consider the following statements financial stability. regarding International Civil Aviation It gives a picture of the resilience of the Organisation (ICAO): financial system. 1. ICAO is an Un Specialized agency, The Report also discusses issues relating to established by states in 1944 to manage the development and regulation of the the administration and governance of financial sector. the Convention on International Civil The report analyses the overall state of the Aviation. (Chicago Convention) various segments as well as highlights the 2. ICAO has empowered to intervene in risk-related issues that could cause potential the airspace of any country. challenges.

www.iasgatewayy.com

UPSC NOVEMBER- 2019 DAILY CURRENT AFFAIRS CLASS TEST - 08 7. Consider the following statements about 9. The Pradyut Bordoloi vs Swapan Roy case Alternative Investment Fund: relates to 1. It is a privately pooled investment (a) Right to privacy vehicle. (b) Office of Profit Guidelines 2. It is regulated by the SEBI. (c) Freedom of Speech and Exression Which of the statements given above is/are (d) Decriminalisation of Homosexuality correct? Answer: b) (a) 1 only Explanation: Supreme Court in Pradyut (b) 2 only Bordoloi vs Swapan Roy (2001) outlined the (c) Both 1 and 2 four broad principles for determining (d) Neither 1 nor 2 whether an office attracts the constitutional Answer: c) disqualification. Explanation: 10. Consider the following statements: AIF means any fund established in India 1. Union Territories are not represented in which is a privately pooled investment vehicle the Rajya Sabha. which collects funds from sophisticated 2. It is within the purview of the Chief investors, whether Indian or foreign, for investing it in accordance with a defined Election Commissioner to adjudicate the investment policy for the benefit of its election disputes. investors. It is regulated by the SEBI. 3. According to the Constitution of India, the Parliament consists of the Lok 8. The Swiss Formula recently in news refers Sabha and the Rajya Sabha only. to Which of the statements given above is/are (a) A mathematical formula designed to cut correct? and harmonize tariff rates in international (a) 1 only trade. (b) 2 and 3 (b) Measuring carbon footprint of industries (c) 1 and 3 (c) Proposed UNSC Seat sharing (d) None methodology Answer: d) (d) None of the above Explanation: None of the above statements Answer: a) are correct Explanation: The Swiss Formula (by the 1. Union Territories (Delhi and Swiss Delegation to the WTO) is a suggested Pondicherry) are represented in the Rajya method for reducing tariff on non- Sabha. agricultural goods (NAMA) by both 2. It is not within the purview of the Chief developed and developing countries. Election Commissioner to adjudicate

www.iasgatewayy.com

UPSC NOVEMBER- 2019 DAILY CURRENT AFFAIRS CLASS TEST - 08 election disputes. It is the Supreme Court and High Court which look into the disputes. 3. According to the Constitution of India, the Parliament consists of the Lok Sabha the Rajya Sabha and the President of India.

www.iasgatewayy.com

UPSC NOVEMBER- 2019 DAILY CURRENT AFFAIRS CLASS TEST - 09 3. Consider the following statements 1. Why ozone hole is formed in Antarctica regarding Moody Ratings to India. instead of Arctic Circle? 1. Moody Ratings are officially provided by 1. Antarctica is colder than Arctic. Niti Aayog of India. 2. More Insolation at Antarctica than 2. AS per the recent ratings, India’s rating Arctic. has been upgraded from Negative to 3. Ratio of land to water is higher in Stable. Antarctica. Choose the correct answer using the code 4. Ratio of land to water is lower in given below: Antarctica. (a) 1 only (a) 1 and 3 only (b) 2 only (b) 2 and 3 only (c) Both 1 and 2 (c) 1 and 4 only (d) Neither 1 nor 2 (d) 2 and 4 only Answer: d) Answer: a) Explanation: Explanation: Moody's Investors Service, often referred to Stratification occurs at colder temperatures as Moody's, is the bond credit rating business and its facilitated better in land compared to of Moody's Corporation. They provide water due to wind dynamics. international financial research on bonds

issued by commercial and government 2. In which of the following river tribunals is entities. So, Statement 1 is incorrect. the government of Karnataka a a party to? Global ratings agency Moody's Investors 1. Cauvery Service has recently cut India's ratings 2. Mandovi / Mahadayi outlook to 'negative' from 'stable' but 3. Godavari affirmed the Baa2 foreign-currency and local- 4. Narmada currency long-term issuer ratings. So, 5. Krishna Statement 2 is incorrect. 6. Vamsadhara Choose the correct answer using the code given 4. Consider the following statements below: regarding ‘Environmental Modification (a) 1,2,3,4 Convention’ which was seen in news (b) 1,2,3,5 recently. (c) 1,3,5,6 1. It aims to prohibit the use of (d) 1,2,4,6 environmental modification techniques Answer: b) as a means of warfare.

www.iasgatewayy.com

UPSC NOVEMBER- 2019 DAILY CURRENT AFFAIRS CLASS TEST - 09 2. India will ratify the convention in Which of the statements given above is/are January 2020 correct? Which of the above statement(s) is / are (a) 1 only correct? (b) 2 only (a) 1 only (c) Both 1 and 2 (b) 2 only (d) Neither 1 nor 2 (c) Both 1 and 2 Answer: c) (d) Neither 1 nor 2 Explanation: Both statements are facts Answer: a) Explanation: 7. Fordow Nuclear Plant, recently seen in India Ratified this Convention in 1978 news is in (a) Iran 5. Recently, volcanic eruption created new (b) Russia island in Tongan archipelago. Its present in (c) North Korea (a) Pacific Ocean (d) China (b) Indian Ocean Answer: a) (c) Atlantic Ocean Explanation: Iran resumed uranium (d) Arctic Ocean enrichment at its underground Fordow Answer: a) nuclear facility. Explanation: Tongan archipelago 8. What is cas9 protein that is often 6. With reference to the Overseas Citizen of mentioned in news? India (OCI), consider the following (a) A molecular scissors used in targeted gene statements: editing. 1. OCI is a person who was a citizen of (b) A biosensor used in the accurate detection of pathogens in patients. India on or after January 26, 1950; or (c) A gene that makes plants pest-resistant was eligible to become a citizen of India (d) An herbicidal substance synthesized in on that date; or who is a child or generally modified crops grandchild of such a person, among Answer: a) other eligibility criteria. Explanation: 2. An applicant is not eligible for the OCI CRISPR/Cas9 DNA editing system was card if he, his parents or grandparents invented six years ago, allowing scientists to have ever been a citizen of Pakistan or modify harmful genes or add new ones. Bangladesh.

www.iasgatewayy.com

UPSC NOVEMBER- 2019 DAILY CURRENT AFFAIRS CLASS TEST - 09

9. With reference to Manipuri Sankirtana, consider the following statements: 1. It is a song and dance performance. 2. Cymbals are the only musical instruments used in the performance. 3. It is performed to narrate the life and deeds of Lord Krishna. Which of the statements given above is/are correct? (a) 1, 2 and 3 (b) 1 and 3 only (c) 2 and 3 only (d) 1 only Answer: b) Explanation: Drums are also used

10. Regarding the International Monetary Fund, which one of the following statements is correct? (a) It can grant loans to any country (b) It can grant loans to only developed countries (c) It grants loans to only member countries (d) It can grant loans to the central bank of a country Answer: c)

www.iasgatewayy.com

UPSC NOVEMBER- 2019 DAILY CURRENT AFFAIRS CLASS TEST - 11 1. ‘Dhrupad’ recently seen in news is referred 4. Ministry of Women and Child to development (a) Defence exercise Select the correct answer using the codes (b) Devotional Music given below: (c) Folk art (a) 1 and 2 only (d) Artillery gun (b) 2 and 3 only Answer: b) (c) 1, 2 and 3 only Explanation: (d) All the above Dhrupad is a form of devotional music that Answer: c) traces its origin to the ancient text of Explanation: Samveda. School Education Quality Index (SEQI) was developed by NITI Aayog to evaluate the 2. Consider the following pairs: performance of States and Union Territories Defence Exercise Country (UTs) in the school education sector. 1. Mitra Shakthi - Sri Lanka Developed through a collaborative process, 2. Gagan Shakthi - Malaysia including key stakeholders such as Ministry 3. Samudra Shakthi - Indonesia of Human Resource and Development Which of the pairs given above is/are correct? (MHRD), the World Bank and sector experts, (a) 1 only the index consists of 30 critical indicators. (b) 1 and 2 only (c) 1 and 3 only 4. In the Context of the Shilpostav- 2019, (d) 1, 2 and 3 consider the following statements: Answer: c) 1. It is the annual fair of artisans from Explanation: across the country, belonging to the Gagan Shakthi is an Indian Air Force weaker sections of the society. exercise. The aim of the exercise was real 2. It is organized by the Union Ministry of time coordination, deployment and Social Justice & Empowerment. employment of air power in a short and Which of the statements given above is/are intense battle scenario. correct? (a) 1 only 3. ‘School Education Quality Index’ is jointly (b) 2 only developed by (c) Both 1 and 2 1. World Bank (d) Neither 1 nor 2 2. NITI Aayog Answer: c) 3. Ministry of Human Resources and Development

www.iasgatewayy.com

UPSC NOVEMBER- 2019 DAILY CURRENT AFFAIRS CLASS TEST - 11 Explanation: While the Nepal government and political parties Shilpostav- 2019 is the annual fair of artisans have protested, India has said the new map does from across the country, belonging to the not revise the existing boundary with Nepal and weaker sections of the society. accurately depicts the sovereign territory of It is organized by the Ministry of Social India. Justice and Empowerment. The artisans assisted by the Apex 6. Consider the following statements with corporations of the Ministry Social Justice respect to Steel Scrap Recycling Policy and Empowerment i.e. National Backward recently released. Classes Finance & Development Corporation 1. It aims to provide standard guidelines (NBCFDC), National Scheduled Caste for collection, dismantling and Finance & Development Corporation shredding activities in an organized, (NSFDC), National Handicapped Finance & safe and environmentally sound manner Development Corporation (NHFDC), 2. Its objective is to promote circular National Safai Karamchari Finance & economy in the steel sector. Development Corporation (NSKFDC) and 3. Scrap based steel making technologies National Trust are displaying and selling their have been envisaged as one of the products. important options to reduce GHG The range of products in this fair include emission intensity. dress materials, leather, cane & bamboo Which of the above statements are correct? products, hand embroidery, bead products, (a) 1 and 2 only block printing, wooden toys, handloom etc. (b) 2 and 3 only Hence only statement 1 is correct. (c) 1 and 3 only

(d) All the above 5. Kalapani, sometimes seen in news, is Answer: d) located in which of the following states of Explanation: Factual statements in the India? policy. The last statement is a point noted in (a) Uttarakhand the National Steel Policy 2017.

(b) Uttar Pradesh (c) Himachal Pradesh 7. Consider the following about National (d) West Bengal Disaster Management Framework in India Answer: a) 1. Ministry of Environment Forest and Explanation: climate change is the Nodal Ministry Kalapani is a 372-sq km area mapped within 2. The National Crisis Management Uttarakhand, bordering far-west Nepal and Committee, headed by PMO is a key Tibet. committee involved in the top-level

www.iasgatewayy.com

UPSC NOVEMBER- 2019 DAILY CURRENT AFFAIRS CLASS TEST - 11 decision-making with respect to among all telecom service areas in terms of Disaster Management. Internet subscriptions per 100population, Which of the above statement(s) is/are behind Delhi, Punjab and Himachal Pradesh. correct (a) 1 only 9. Recently being debated - The Triffin (b) 2 only dilemma or Triffin paradox referes to (c) 1 and 2 (a) Development vs Environment Question (d) Neither 1 nor 2 (b) The conflict of economic interests that Answer: d) arises between short-term domestic and Explanation: Ministry of Home Affairs is long-term international objectives for the Nodal Ministry countries whose currencies serve as global The National Crisis Management Committee reserve currencies. is headed by Cabinet secretary. (c) WTO rounds of negotiations on Rules of Origin 8. According to Internet and Mobile (d) None of the above Association of India's (IAMAI) report, titled Answer: b) ‘India Internet 2019’, which of the following Explanation: The Triffin dilemma or Triffin State has least Internet Penetration in paradox is the conflict of economic interests India. that arises between short-term domestic and (a) Odisha long-term international objectives for (b) Jharkhand countries whose currencies serve as global (c) Bihar reserve currencies. (d) Utter Pradesh Answer: a) Explanation: According to Internet and Mobile Association of India (IAMAI), report, titled ‘India Internet 2019’, Delhi NCR has the highest internet penetration (69%) followed by Kerala (54%). The lowest penetration rate was in Odisha (25), Jharkhand (26) and Bihar (28). The IAMAI report also notes that Kerala, Tamil Nadu and Delhi have the highest proportion of female Internet users. Further, TRAI data as of June 2019 show that Kerala is well-placed in terms of Internet connections. It stands fourth

www.iasgatewayy.com

UPSC NOVEMBER- 2019 DAILY CURRENT AFFAIRS CLASS TEST - 11 10. Sunni and Shia Waqf boards of UP were in news recently. In that context, which of the following statements are correct about the Central Waqf Council? 1. It is a statutory body under the administrative control of the Ministry of Minority Affairs. 2. It can render legal advice on protection and retrieval of the Waqf Properties and for removal of encroachment etc. Which of the above statement(s) is/are incorrect? (a) 1 only (b) 2 only (c) Both 1 and 2 (d) Neither 1 nor 2 Answer: d) Explanation: Both statements are correct

www.iasgatewayy.com

UPSC NOVEMBER- 2019 DAILY CURRENT AFFAIRS CLASS TEST - 12 1. Consider the following statements. (d) None of the above 1. India has Free Trade Agreements with Answer: d) all the RCEP members. Explanation: 2. India has free Trade Agreement with all Born on 11 November 1888 and died on 22 ASEAN members in both goods and February 1958. He was senior Muslim leader services. of Indian National Congress during Indian Which of the above statement(s) is/are correct? independence movement. In 1923, at age of (a) 1 only 35, he became youngest person to serve as (b) 2 only President of Indian National Congress. He (c) Both 1 and 2 was first Minister of Education of (d) Neither 1 nor 2 Independent India. The weekly Urdu Answer: b) language newspaper ‘Al-Hilal’ was Explanation: Currently, RCEP consists of established by him. 10 ASEAN (Association of South-East Asian Nations) governments and their five FTA 3. Choose the correct statement regarding partners: Australia, China, Japan, New 'Swiss challenge' method Zealand and South Korea. (a) It is a method of bidding, often used in India has free trade agreements with all the public projects. members of RCEP except China, (b) In this method, a third parties make offers Australia and New Zealand. (challenges) for a project within a India and ASEAN signed the FTA in goods in designated period to avoid exaggerated 2009. India also signed an FTA in project costs. services and investments with ASEAN in (c) Governments in India had implemented 2014, which came into force in 2015. various projects under Swiss Challenge Method 2. Consider the following statements (d) All of the above. regarding Maulana Abul Kalam Azad Answer: d) 1. He was the youngest President of INC. Explanation: 2. He was first Education Minister of the A Swiss Challenge is a method of bidding, often used in public projects, in which an Independent India. interested party initiates a proposal for a 3. The weekly Urdu language newspaper contract or the bid for a project. The ‘Al-Hilal’ was established by him. government then puts the details of the Which of the following are incorrect? project out in the public and invites proposals (a) Only (1) and (2) from others interested in executing it. On the (b) Only (2) and (3) (c) Only (1) and (3) www.iasgatewayy.com

UPSC NOVEMBER- 2019 DAILY CURRENT AFFAIRS CLASS TEST - 12 receipt of these bids, the original contractor Answer: C) gets an opportunity to match the best bid. Explanation: The Swiss Challenge allows a seller to mix- As per Sikh tradition, Nanak was a blessed or and-match the features of both an open illuminated soul. He is not considered an auction and a closed tender to discover the incarnation or even a prophet. He rejected best price for an asset. the authority of Vedas. He initiated the institution of Guru ka Langar. He was the 4. Internet Freedom Report 2019 is released contemporary of Babur. by The Khalsa, the saint-soldier order was (a) Economic Intelligence Unit created by Guru Gobind Singh. (b) International Telecommunication Union (c) Human Development Internationale 6. Consider the following about Zero Carbon (d) Freedom House, a Non-Government Law, recently passed by New Zealand. Organization 1. This is the first legislation in the world Answer: d) to make a legally binding commitment Explanation: Freedom House is a U.S.- to living within 1.5 degrees Celsius of based non-governmental organization (NGO) global warming. that conducts research and advocacy on 2. The Act proposes separate targets for democracy, political freedom, and human biogenic methane rights. The Freedom on the Net reports Which of the above statement(s) is/are provide analytical reports and numerical correct? ratings regarding the state of Internet (a) 1 only freedom for countries worldwide (b) 2 only

(c) Both 1 and 2 5. Choose the correct statement regarding (d) Neither 1 nor 2 ‘The First Sikh Guru’ Answer: c) 1. Guru Nanak started the institution of Explanation: New Zealand’s Parliament common kitchen (Guru ka Langar) passed The Zero-Carbon Act, which will 2. He was the contemporary of Babur. commit New Zealand to zero carbon 3. He rejected the authority of the Vedas. emissions by 2050 or sooner, as part of the 4. He created the Khalsa, a saint – soldier country’s attempts to meet its Paris climate order. accord commitments. The Act is not a (a) Only (1) and (2) separate legislation but is an amendment to (b) Only (1) and (3) the existing Climate Change Responses Act, (c) Only (1), (2) and (3) 2002.The key aims of the Act include: reduce (d) All of the above. all greenhouse gases (except methane) to net

www.iasgatewayy.com

UPSC NOVEMBER- 2019 DAILY CURRENT AFFAIRS CLASS TEST - 12 zero by 2050, reduce emissions of biogenic 8. Consider the following statements methane (produced from biological sources) regarding the Jal Jeevan Mission (JJM): up to 24-47 percent below 2017 levels by 1. It aims at providing Functional 2050 and to 10 percent below 2017 levels by Household Tap Connections (FHTCs) in 2030, establish an independent Climate rural areas by 2024. Change Commission and establish a system 2. Department of Drinking Water & of emissions budget. Sanitation is the implementing

Department for the mission. 7. ‘Pliosaur’, recently seen in news refers to Which of the statements given above is/are (a) a deadly virus in East Africa correct? (b) A Spyware developed by Israel (a) 1 only (c) a fungal infection in plants (b) 2 only (d) Predatory reptiles that lived million years (c) Both 1 and 2 ago (d) Neither 1 nor 2 Answer: d) Answer: c) Explanation: Explanation: Pliosaur were a group of large carnivorous The Jal Jeevan Mission (JJM) aims at marine reptiles characterized by massive providing Functional Household Tap heads, short necks, and streamlined tear- Connections (FHTCs) in rural areas by 2024. shaped bodies. It was announced by the Prime Minister; Pliosaurs have been found as fossils from the Narendra Modi on 73rd Independence Day, Jurassic and Cretaceous periods (about 200 Government of India and State Governments million to 65.5 million years ago). They are will work in partnership to further this goal. classified in the order Plesiosauria, along with Approx. their long-necked relatives, the plesiosaurs. 14.60 crores rural households of the country They were the largest aquatic carnivorous will be provided with FHTCs. The provisional reptiles that have ever lived and are often cost estimate for this mission is Rs 3.60 lakh dubbed “sea monsters”. They measured over Crores. 10 metres in length and could weigh up to Department of Drinking Water & Sanitation several dozen tons. is the implementing Department for the They had powerful, large skulls and massive mission. jaws with large, sharp teeth. Their limbs were

in the form of fins. They used four large fins

to swim through Mesozoic seas.

www.iasgatewayy.com

UPSC NOVEMBER- 2019 DAILY CURRENT AFFAIRS CLASS TEST - 12 9. Rajiv Gauba Committee which was recently Maitreya is the earliest bodhisattva around in news is related to? whom a cult developed and is mentioned in (a) To mitigate financial stress in telecom scriptures from the 3rd century CE. He was sector accepted by all schools of Buddhism and is (b) Train ticket reservations still the only bodhisattva generally honoured (c) Reforms in Banking sector by the Theravada tradition. (d) Reforms in the GST Council Answer: a) Explanation: The government has set up a Committee of Secretaries (CoS) under Cabinet Secretary Rajiv Gauba to suggest measures to mitigate financial stress in the telecom sector. The CoS will comprise representatives from the Ministries of Finance, Law and Telecom. Sources in the government said it will consider issues such as deferment of spectrum auction for 2 years and reduction in spectrum usage charges, and take a relook at the Universal Service Obligation Fund (USOF) charges.

10. With reference to Indian history, who among the following is a future Buddha, yet to come to save the world? (a) Avalokiteshvara (b) Lokesvara (c) Maitreya (d) Padmapani Answer: c) Explanation: Maitreya, in Buddhist tradition, the future Buddha, presently a bodhisattva residing in the Tushita heaven, who will descend to earth to preach a new the dharma (“law”) when the teachings of Gautama Buddha have completely decayed.

www.iasgatewayy.com

UPSC NOVEMBER- 2019 DAILY CURRENT AFFAIRS CLASS TEST - 13 1. Choose the incorrect statement regarding Choose the correct statement: ‘Model Code of Conduct’ (a) Only 1 and 2 (a) The MCC is a set of norms for conduct (b) Only 1 and 3 and behaviour on the part of the Parties (c) All of the above and candidates (d) Only 2 and 3 (b) It is framed without the consensus of the Answer: c) political parties, solely by the Election commission. 3. Arrange the following from North to South (c) It is not enforceable by law 1. Golan Heights (d) The MCC remains in force from the date 2. Gaza Strip of announcement of elections till the 3. Sinai Peninsula completion of elections. (a) 3-1-2 Answer: b) (b) 1-3-2 Explanation: MCC are set of guidelines (c) 2-1-3 issued by ECI to regulate political parties and (d) 1-2-3 candidates prior to elections. It ensures free Answer: d) and fair elections. The MCC will be in place Explanation: Map Based question from announcement of the election schedule by the Election Commission, till the date of 4. Consider the following statements about results. MCC is not legally binding, it does not Minamata Convention. have any separate statutory backing. 1. India has signed and will ratify However, it can be enforced through certain Minamata Convention in January 2020 other provisions of like Representation of 2. There is no monitoring Programme in People Act and IPC. MCC is framed with the India to regulate export and import of consensus of the political parties. Mercury.

2. Consider the following regarding Air Which of the above statements are incorrect? (a) 1 only Quality Index (b) 2 only 1. It focuses on health effects one might (c) Both 1 and 2 experience within a few hours or days (d) Neither 1 nor 2 after breathing polluted air. Answer: c) 2. AQI is calculated for eight major air Explanation: India signed the Convention pollutants - Ground-level ozone, PM10, in 2014 and ratified in 2018. PM2.5, CO, SO2, NO2, NH3 and Lead. In India, it is regulated by Hazardous Wastes 3. The AQI has six colour codes to indicate (Management and Handling) Rules, 1989 level of pollution in cities.

www.iasgatewayy.com

UPSC NOVEMBER- 2019 DAILY CURRENT AFFAIRS CLASS TEST - 13 amended in 2003, which prohibits the import interoperability for conducting HADR and export of mercury-bearing waste and operations. stipulate environmentally sound 7. Consider the following about Bushehr management.

Nuclear Power Plant. 5. Consider the following statements about 1. It is the first civilian nuclear power plant Places of Worship (Special Provisions) Act built in the Middle East. 1991. 2. Its located along the Persian Gulf coast 1. It prohibits conversion of any place of of south-western Iran. worship Which of the above statements are correct? 2. Provide for the maintenance of the (a) 1 only religious character of any place of (b) 2 only worship as it existed on the 26th January (c) Both 1 and 2 1950. (d) Neither 1 nor 2 Which of the above statements are correct? Answer: c) (a) 1 only Explanation: Both are actual statements (b) 2 only (c) Both 1 and 2 8. In the context of Foreign Contribution (d) Neither 1 nor 2 Regulation Act (FCRA) 2010, consider the Answer: a) following statements: Explanation: Provide for the maintenance 1. Under this act, candidate contesting the of the religious character of any place of election is debarred from receiving worship as it existed on the 15th day of contribution. August 1947. 2. FCRA comes under the purview of the

Department of Economic Affairs of 6. Recently in news, "Tiger Triump" is (a) India - China Tiger Conservation Effort Ministry of Finance. (b) Cryogenic technology deployed by ISRO Which of the statement(s) given above is/are (c) Maiden India US joint Tri services correct? Humanitarian Assistance and Disaster (a) 1 only Relief (HADR) Exercise. (b) 2 only (d) None of the above (c) Both 1 and 2 Answer: c) (d) Neither 1 nor 2 Explanation: ‘Tiger Triumph’ is the maiden Answer: a) India US joint Tri services Humanitarian Assistance and Disaster Relief (HADR) Exercise. The Exercise is aimed to developing www.iasgatewayy.com

UPSC NOVEMBER- 2019 DAILY CURRENT AFFAIRS CLASS TEST - 13

Explanation: Answer: c) As per FCRA 2010, the following Explanation: organisational individuals are debarred from NAM is an online platform with a physical receiving foreign contribution. They are market or mandi at the backend. NAM is not 1. Candidate for election a parallel marketing structure but rather an 2. cartoonist, editor, publishers of registered instrument to create a national network of newspaper physical mandis which can be accessed 3. Judge, government servants or employee online. of any corporation It seeks to leverage the physical 4. Member of any legislature infrastructure of mandis through an online 5. Political parties. trading portal, enabling buyers situated even Despite being a law related to financial outside the state to participate in trading at regulation, this law does not fall within the the local level. purview of the RBI but under the Home The launching of e-platform for marketing of Ministry as it is internal security legislation. agriculture products is being done with the aim to provide more options to farmers to sell 9. Consider the following statements their produce. regarding the electronic National This initiative is part of implementation of Agriculture Market (e-NAM): the roadmap for doubling income of the 1. NAM is a parallel marketing structure farmers by 2022. and an instrument to create a national Since its introduction in 2016, only 1.6 crore network of physical mandis which can farmers have registered on the portal so far, be accessed online. from among the almost 12 crore cultivators in 2. The launching of e-platform for the country. According to data presented in marketing of agriculture products is the Lok Sabha in June, only about half of being done with the aim to provide those registered have benefited from the platform. more options to farmers to sell their

produce.

3. Farmers have actively involved in e-

NAM since its introduction in 2016.

Which of the statement(s) given below is/are

incorrect?

(a) 1 only

(b) 3 only

(c) 1 and 3 only (d) 2 and 3 only www.iasgatewayy.com

UPSC NOVEMBER- 2019 DAILY CURRENT AFFAIRS CLASS TEST - 13 10. A community of people called Manganiyars is well known for their (a) Martial arts in North-East India (b) Musical tradition in North-West India (c) Classical vocal music in South India (d) Pietra dura tradition in Central India Answer: b) Explanation: The Manganiar and related Langha caste are Muslim communities in the desert of Rajasthan, India in the districts of Barmer and Jaisalmer, along the border with Pakistan. Significant numbers are also found in the districts of Tharparkar and Sanghar in the province of Sindh in Pakistan. They are famous for their classical folk music.

www.iasgatewayy.com

UPSC NOVEMBER- 2019 DAILY CURRENT AFFAIRS CLASS TEST - 14 1. ‘Dampier Hodges Line’ recently seen in has initiated the car rationing scheme which news is related to is also called as odd-even scheme. (a) Gulf of Kutch Private vehicles will be allowed to run across (b) Palk Strait the city based on their registration numbers. (c) Andaman and Nicobar Islands For example, if a vehicle’s registration (d) Sunderbans number ends with an odd digit, it will be Answer: d) allowed on the road on January 1, while that Explanation: ending with an even number can be driven on The Indian Sunderbans, considered to be an the second, and so on. area south of the Dampier Hodges line, is spread over 9,630 sq. km., of which the 3. In the context of Collegium system, mangrove forests are spread over 4,263 sq. consider the following statements: km. 1. The Collegium System is a system under which appointments of judges of High 2. Consider the following statements Courts and Apex Court are decided by a regarding the odd-even scheme: forum. 1. Under the scheme, private vehicles will 2. There is no mention of the Collegium be allowed to run across the city either in the original Constitution of alternatively based on their registration India or in successive amendments. numbers. 3. The recommendations of the Collegium 2. It has been implemented on the light of are binding on the Central Government. increasing pollution in the city. Which of the statements given above are 3. Union Government has implemented correct? this scheme across North India (a) 1 and 2 only including the National capital territory (b) 2 and 3 only Delhi. (c) 1 and 3 only Which of the statement(s) given above is/are (d) 1, 2 and 3 correct? Answer: d) (a) 1 only Explanation: (b) 1 and 2 only All the statements are facts about the (c) 1 and 3 only Collegium system. (d) 1, 2 and 3

Answer: b)

Explanation:

To control the effects of pollution in the

National Capital territory, Delhi government

www.iasgatewayy.com

UPSC NOVEMBER- 2019 DAILY CURRENT AFFAIRS CLASS TEST - 14 4. With respect to the Tribunals which of the legally binding on the parties, subject to statement(s) given below is/are incorrect? appeal. 1. A tribunal is a quasi-judicial body established an Act of Parliament or 5. Global Competitiveness Index is an annual State Legislature under Article 323A or report compiled by? (a) World Economic Forum 323B of the Indian Constitution. (b) World Trade Organisation 2. It is a forum where cases pending in the (c) UN Conference on Trade and court of law or at pre-litigation stage are Development settled or compromised amicably. (d) World Bank 3. Its decisions are legally binding on the Answer: a) parties, subject to appeal. Explanation: Select the Correct answer using the codes Global Competitiveness Index is the annual given below: report compiled by Geneva-based World (a) 1 only Economic Forum (WEF). (b) 1 and 2 only

(c) 2 only 6. ‘Sagar Island’ sometimes seen in news is (d) 1 and 3 only located in which of the following Indian Answer: c) States? Explanation: (a) West Bengal 2nd statement is related to the Lok Adalat. (b) Odisha A tribunal is a quasi-judicial body established (c) Gujarat in India by an Act of Parliament or State (d) Maharashtra Legislature under Article 323A or 323B to Answer: a) resolve disputes that are brought before it. Explanation: They play an important role and part in the Cyclone Bulbul recently made its landfall in sphere of the adjudication of disputes the Sagar Island of West Bengal. especially when the subject demands

technical expertise. 7. Consider the statements They do not have to follow any uniform (a) The president can appoint a HC judge as procedure as laid down under the Civil acting chief justice in case the vacancy of Procedure Code and the Indian Evidence Act office of chief justice or when the CJ is but they have to follow the principles of temporarily absent or unable to perform Natural Justice. his duties. They enjoy some of the powers of a civil court, viz., issuing summons and allowing witnesses to give evidence. Its decisions are www.iasgatewayy.com

UPSC NOVEMBER- 2019 DAILY CURRENT AFFAIRS CLASS TEST - 14 (b) The CJI with prior consent of the (d) All the above President and the CJ of the HC can Answer: d) appoint ad-hoc judges. 9. For the Karachi session of Indian National (c) Both (a) and (b) Congress in 1931, presided over by Sardar (d) None of the above. Patel, who drafted the Resolution on Answer: c) Explanation: Fundamental Rights and Economic Appointment of Ad hoc Judges (Article 127): Programme? If at any time there is a lack of quorum of the (a) Mahatma Gandhi Judges of the Supreme Court to hold or (b) Pandit Jawaharlal Nehru continue any session in the Court, the Chief (c) Dr. Rajendra Prasad Justice of India may with the prior consent of (d) Dr. B. R. Ambedkar the President and after consultation with the Answer: b) Chief Justice of the High Court concerned, Explanation: request in writing the attendance at the Some important aspects of the Karachi sittings of the Court, as an ad hoc Judge, for session of resolutions were: Basic civil rights such period as may be necessary of freedom of speech, Freedom of Press, Appointment of acting Chief Justice (Article Freedom of assembly, Freedom of 126): When the Office of the Chief Justice of association, Equality before law Elections on India is vacant or when the Chief Justice, by the basis of Universal Adult Franchise Free reason of absence or otherwise, is unable to and compulsory primary education. perform the duties of his office, the duties of Substantial reduction in rent and taxes Better the office shall be performed by such one of conditions for workers including a living the other Judges of the Court as the President wage, limited hours of work. Protection of may appoint for the purpose. women and peasants Government ownership or control of key industries, mines, and 8. Choose the correct statement transport. Protection of Minorities. Sardar (a) The 2019 Brown to Green Report is Vallabhai Patel presided over the Karachi published by the Climate Transparency session of Indian National Congress in 1931. partnership. The resolutions adopted were: (b) The report noted that Carbon emissions  Fundamental Right and Economic Plan from the world’s 20 biggest economies, Programme were formulated by Jawaharlal including India, are rising. Nehru and adopted in this session. (c) According to the report, India ranked fifth  It admitted Gandhi-Irwin Pact as Gandhi highest among G20 countries in terms of was allowed to attend the second Round deaths from extreme weather events Table Conference in London.

www.iasgatewayy.com

UPSC NOVEMBER- 2019 DAILY CURRENT AFFAIRS CLASS TEST - 14 10. Recently launched forestry project, Capacity-building Initiative for Transparency (CBIT) is an initiative of: (a) NABARD (b) Global Environment Facility (c) United Nation Development Programme (d) Global Climate Fund Answer: b) Explanation: The Capacity-building Initiative for Transparency (CBIT) supports developing countries to build institutional and technical capacity, both pre- and post- 2020, to meet enhanced transparency requirements as defined in Article 13 of the Paris Agreement.

www.iasgatewayy.com

UPSC NOVEMBER- 2019 DAILY CURRENT AFFAIRS CLASS TEST - 15 1. Which of the following statements is/are 2. Which of the following statements is/are correct? incorrect? 1. Type 1 diabetes mellitus is a chronic 1. Pamba River is the one of the longest condition in which the pancreas rivers in the Kerala. produces little or no insulin. 2. It originates in Western Ghats and then 2. Type 2 diabetes mellitus is a chronic flows west before emptying into the condition that affects the way the body Vembanad Lake. processes blood sugar (glucose) 3. Sabarimala temple is located on the 3. Type 2 diabetes is also called as non- banks of the river Pamba. insulin-dependent diabetes (NIDD). 4. The river is also known as 'Dakshina 4. Type 1 diabetes is also called insulin- Bhageerathi' dependent diabetes. Select the correct answer using the codes Select the correct answer using the codes given below: given below: (a) 1 and 2 only (a) 1 and 2 only (b) 1, 2 and 3 only (b) 1, 3 and 4 only (c) 4 only (c) 1, 2 and 3 only (d) None of the above (d) 1,2,3 and 4. Answer: d) Answer: d) Explanation: Explanation: Pampa River is the third longest river in the Type 1 diabetes is also called insulin- Kerala after Periyar and Bharathappuzha. dependent diabetes and it is believed to be an The river is also known as 'Dakshina autoimmune condition. Bhageerathi'. It originates at Pulachimalai hill In Type 1, body’s immune system mistakenly of the Western Ghats in Kerala and then attacks and destroys the beta cells in the flows west before emptying into the pancreas that produce insulin. Vembanad Lake. Sabarimala temple Type 2 diabetes is also called as non-insulin- dedicated to Lord Ayyappa is located on the dependent diabetes (NIDD), leads to a banks of the river Pamba. condition where one’s body cannot use the insulin efficiently. 3. Consider the following statements This in turn affects the way the body regarding Diabetes Atlas 2019: processes blood sugar (glucose). 1. It is a biennial report developed by the World Health Organisation (WHO) released on the occasion of International Diabetes Day.

www.iasgatewayy.com

UPSC NOVEMBER- 2019 DAILY CURRENT AFFAIRS CLASS TEST - 15 2. It stated that India has the second The number of people with diabetes is highest diabetes patients after China. predicted to rise to 578 million by 2030 and Which of the statement(s) given above is/are to 700 million by 2045. correct? 374 million adults have impaired glucose (a) 1 only tolerance, placing them at high risk of (b) 2 only developing type 2 diabetes. (c) Both 1 and 2 (d) Neither 1 nor 2 4. Sabarimala temple is located in which of Answer: b) the following tiger reserves? Explanation: (a) Periyar Tiger reserve The International Diabetes Foundation (IDF) (b) Parambikulam Tiger reserve Diabetes Atlas is the authoritative resource (c) Anaimalai Tiger reserve on the global burden of diabetes. (d) Mudhumalai Tiger reserve First published in 2000, it is produced by Answer: a) International Diabetes Foundation (IDF) Explanation: biennially in collaboration with experts from The Sabarimala temple is a temple complex around the world and contains data on located at Sabarimala inside the Periyar Tiger diabetes cases, prevalence, mortality and Reserve in Pathanamthitta district, Kerala, expenditure on the global, regional and India. national level. 5. Consider the following statements A full IDF Diabetes Atlas report is produced regarding the Right to Information (RTI) every two years. Act, 2005: India continues to be home to the second- 1. RTI is applicable to the whole of India largest number of adults with diabetes including all the Union territories. worldwide, with 77 million adults with diabetes in the 20-79 years age group. This 2. Central Armed Police Force (CAPF) is follows China, which has 116 million adults completely exempted from RTI. with diabetes in the same age profile. 3. Recently the office of the Chief Justice The worldwide prevalence of diabetes was of India (CJI) also came under the RTI’s estimated at 463 million in this age group, or ambit. in other words, one in 11 adults. Which of the statement(s) given above is/are India was the largest contributor to diabetes correct? mortality with more than 1 million estimated (a) 1 only deaths attributable to diabetes and related (b) 2 and 3 only complications, in the larger South East Asian (c) 1 and 3 only region. (d) 1, 2 and 3

www.iasgatewayy.com

UPSC NOVEMBER- 2019 DAILY CURRENT AFFAIRS CLASS TEST - 15 Answer: c) 7. Which of the following are wrongly Explanation: Matched? The Official Secrets act, 1923 over rides the 1. Pradhan Mantri Shram Yogi Maan- RTI acts 2005. If any information the public Dhan (PM-SYM) - old age protection authority finds it sensitive or it concerns the and social security of Unorganized national security by any means or it falls Workers under the purview of the OSA, 1923 then it is 2. Pradhan Mantri Kisan Samman Nidhi exempted. (PM-KISAN) - all small and marginal Government agencies and bodies like CAPF, farmers will get up to ₹6,000 per year CBI, NIA, RAW does not falls under RTI Act. as minimum income support Any information concerning personal 3. PM Kisan Maan Dhan Yojana - old age information of any individual, like salary, pension scheme for all land holding account statement is also exempted. Small and Marginal Farmers (SMFs) in These organisations are exempted under RTI the country Act, but not completely exempted as cases (a) 1 only related to corruption and Human Rights (b) 1 and 3 only Violation done by them is not exempted. (c) 1,2 and 3

6. ‘World Vision Report’ which was released (d) None of the above recently by which of the following agencies? Answer: d) Explanation: All the statements are correct (a) UNICEF

(b) Doctors without Borders (c) World Health Organisation 8. Arrokoth recently seen in news refers to, (d) World Bank (a) Newly discovered Banana species in Answer: c) Andaman Explanation: (b) Indigenously built Bofors varient The World Health Organization (WHO) (c) An icy body in Kuiper Belt released its first World Vision Report (d) None of the above recently, proposing ways to address Answer: c) challenges such as integrating eye care into Explanation: It is Ultima Thule's new name healthcare systems. The report found that globally, over 2.2 billion people have vision impairment. The report also said that out of these 2.2 billion, 1 billion people are suffering from conditions that are preventable, or unaddressed.

www.iasgatewayy.com

UPSC NOVEMBER- 2019 DAILY CURRENT AFFAIRS CLASS TEST - 15 9. Recently discovered species, Marengo Sachin Tendulkar is a: (a) Snake (b) Frog (c) Fish (d) Spider Answer: d) Explanation: Dhruv Prajapati, a junior researcher with the Gujarat Ecological Education and Research (GEER) Foundation, pursuing a PhD in spider taxonomy, discovered two new species of the arthropod and after naming one after Saint Kuriakose Elias Chavara, who played a key role in creating awareness on education in Kerala, named another after Sachin Tendulkar

10. Gaza Strip’ sometimes seen in news is located along the coast of (a) Red Sea (b) Mediterranean Sea (c) Black Sea (d) Caspian Sea Answer: b) Explanation: The Gaza Strip or simply Gaza, is a self- governing Palestinian territory on the eastern coast of the Mediterranean Sea, that borders Egypt on the southwest for 11 kilometres (6.8 mi) and Israel on the east and north along a 51 km (32 mi) border. Gaza and the West Bank are claimed by the State of Palestine.

www.iasgatewayy.com

UPSC NOVEMBER- 2019 DAILY CURRENT AFFAIRS CLASS TEST - 16 1. Which of the following statements is/are Nominated members cannot vote in the correct? election of the President, however, they can 1. Rajya Sabha also can initiate the participate in President’s impeachment. impeachment charges against President.

2. The only condition for the initiation of 2. Consider the following statements impeachment of Indian president is the regarding Sikhism ‘violation of the constitution.’ 1. Guru Gobind Singh, compiled the first 3. Nominated members cannot vote in the official edition of the Sikh scripture election of the President, however, they called the Adi Granth. can participate in President’s 2. Guru Arjan Das, introduced the Five Ks impeachment. (Kesh, Kangha, Kara, Kirpan and Select the correct answer using the codes Kacchera), the five articles of Khalsa given below faith. (a) 1 and 2 only 3. Guru Ram Das was the founder the city (b) 1, 2 and 3 only of Amritsar and started the construction (c) 1 and 3 only of the famous Golden Temple at (d) 3 only Amritsar. Answer: b) Select the incorrect using the codes given Explanation: ARTICLE 61 gives the Below: Procedure for Impeachment of The President (a) 1 and 2 only When a President is to be impeached for (b) 1, 2 and 3 only violation of the Constitution, the charge shall (c) 1 and 3 only be preferred by either House of Parliament. (d) None of the above No such charge shall be preferred unless – Answer: a) resolution has to be passed after at least Explanation: Gurus in statement 1 and 2 fourteen days’ notice in writing signed by not are Interchanged. less than one-fourth of the total number of

members of the House 3. River Sisar(Sisiri) is a major tributary of such resolution has to be passed by a majority (a) Dibang of not less than two-thirds of the total (b) Yamuna membership of the House. (c) Pamba The President shall have the right to appear (d) Godavari and to be represented at such investigation. Answer: a) The only condition for the initiation of Explanation: Defence Minister Rajnath impeachment of Indian president is the Singh inaugurated the Sisiri River Bridge in ‘violation of the constitution. www.iasgatewayy.com

UPSC NOVEMBER- 2019 DAILY CURRENT AFFAIRS CLASS TEST - 16 East Siang District and also said that the Answer: b) North Eastern Region is very crucial for the Explanation: nation from a strategic point of view. The According to The Asia Pacific Trade and Sisar(Sisiri), Mathun, Tangon, Dri, Ithun and Investment Report 2019, non-tariff measures Emra are the major tributaries of the Dibang. (NTMs) have increased in the past two decades and are affecting trade as well 4. Recently in news "NISHTHA" refers to sustainable development goals (SDGs) in (a) Initiative Holistic Advancement for Asian countries. School Heads and Teachers It was published by the United Nations (b) Newly discovered species of Snake in Economic and Social Commission for Asia Western Ghats and the Pacific (ESCAP) and the United (c) Icy region in outer space Nations Conference on Trade and (d) None of the above Development (UNCTAD). Answer: a) Explanation: NISHTHA is a capacity 6. Consider the following statements building programme for "Improving Quality regarding the Tenth Schedule of the Indian of School Education through Integrated constitution: Teacher Training" by Ministry of Human 1. A person disqualified under the Tenth Resource Development. NISHTHA is the Schedule is not eligible to contest in re- largest teachers’ training programme of its election to that constituency. kind in the world. 2. As per the existing constitutional

mandate, the Speaker is not empowered 5. Asia-Pacific Trade and Investment Report to disqualify any member for the entire 2019 is jointly published by? term of the House. 1. U.S. Chamber of Commerce Which of the statement(s) given above is/are 2. UN Conference on Trade and correct? Development (a) 1 only 3. UN Economic and Social Commission (b) 2 only for Asia and the Pacific (c) Both 1 and 2 Select the correct answer using the codes (d) Neither 1 nor 2 given below: Answer: b) (a) 1 and 2 only Explanation: (b) 2 and 3 only The Supreme Court has recently upheld the (c) 1 and 3 only disqualification of 17 dissident by then (d) 1, 2 and 3 Karnataka Assembly Speaker under the Tenth

www.iasgatewayy.com

UPSC NOVEMBER- 2019 DAILY CURRENT AFFAIRS CLASS TEST - 16 Schedule (anti-defection law) but held that 8. Bolivia is bordered by which of the their ouster was no bar to contesting re-polls. following countries? Neither under the Indian Constitution nor 1. Chile under the statutory scheme it is contemplated 2. Peru that disqualification under the Tenth 3. Brazil Schedule would operate as a bar for 4. Paraguay contesting re-elections. 5. Argentina In the light of the existing constitutional Select the correct answer using the codes mandate, the Speaker is not empowered to given below: disqualify any member till the end of the (a) 1 and 2 only term. (b) 1, 2 and 3 only

7. Tiger reserves are absent in which of the (c) 1, 3, 4 and 5 only (d) All the above following Indian state? Answer: d) 1. Bihar Explanation: 2. Uttar Pradesh A landlocked country, Bolivia shares borders 3. Sikkim with Brazil to the north and east, Paraguay to 4. Haryana the southeast, Argentina to the south, Chile to Select the correct answer using the codes the southwest, and Peru to the northwest. given below:

(a) 1 and 2 only 9. Consider the following statements (b) 2 and 3 only regarding the National Company Law (c) 3 and 4 only Appellate Tribunal (NCLAT): (d) 1 and 4 only 1. It is a quasi-judicial body constituted Answer: c) under the Section 410 of the Companies Explanation: Tiger reserves are absent in Indian states of Act, 2013. Punjab, Haryana, Goa, Gujarat, Sikkim, 2. The President, chairperson and Judicial Meghalaya, Tripura, Manipur and Nagaland. Members of the Appellate Tribunal are Tiger reserves are also absent in all other appointed after consultation with the Union territories such as Jammu & Kashmir, President of India. Ladakh, Puducherry, Chandigarh, Delhi, 3. It can only hear and dispose appeals Andaman & Nicobar, Lakshadweep, Dadra & against any orders passed by the Nagar Haveli, Daman & Diu. Competition Commission of India (CCI). Which of the statement(s) given below is/are incorrect?

www.iasgatewayy.com

UPSC NOVEMBER- 2019 DAILY CURRENT AFFAIRS CLASS TEST - 16 (a) 2 only 10. Recently in news, Acqua alta refers to (b) 2 and 3 only (a) High tides in the Adriatic Sea (c) 3 only (b) Low tides regions of Pacific Ocean (d) 1, 2 and 3 (c) High tides in Black Sea Answer: b) (d) Low tides regions of Caspian Sea Explanation: Answer: a) The President, chairperson and Judicial Explanation: High tides in the Adriatic Sea Members of the Appellate Tribunal are - (Acqua alta) have caused floods in the appointed after consultation with the Chief historic city of Venice. Justice of India. National Company Law Appellate Tribunal (NCLAT) was constituted under Section 410 of the Companies Act, 2013 for hearing appeals against the orders of National Company Law Tribunal(s) (NCLT), with effect from 1st June, 2016. NCLAT is also the Appellate Tribunal for hearing appeals against the orders passed by NCLT(s) under Section 61 of the Insolvency and Bankruptcy Code, 2016 (IBC). NCLAT is also the Appellate Tribunal for hearing appeals against the orders passed by Insolvency and Bankruptcy Board of India under Section 202 and Section 211 of IBC. NCLAT is also the Appellate Tribunal to hear and dispose of appeals against any direction issued or decision made or order passed by the Competition Commission of India (CCI).

www.iasgatewayy.com

UPSC NOVEMBER- 2019 DAILY CURRENT AFFAIRS CLASS TEST - 18 1. Consider the following statements partnerships, limited liability partnerships, regarding the Insolvency and Bankruptcy individuals etc. Code: 1. Bankruptcy is the situation where the 2. In the context of Coral Spawning, consider debtor is not in a position to pay back the following statements: the creditor while insolvency is the legal 1. Coral Spawning is one of the annual declaration of bankruptcy. synchronised events to occur on the 2. The Insolvency and Bankruptcy Code Great Barrier Reef. can be triggered if there is a minimum 2. It occurs by releasing tiny egg and default of Rs 1 lakh. sperm bundles simultaneously from 3. The code will apply to all sorts of their gut cavity into the water. business entities including corporate 3. The spawning takes place on a large companies, partnerships, and limited scale as it happens across the entire liability partnerships except the Reef all at once. individuals. Which of the statements given above are Which of the statement(s) given above is/are correct? incorrect? (a) 1 and 2 only (a) 1 only (b) 2 and 3 only (b) 1 and 2 only (c) 1 and 3 only (c) 3 only (d) 1, 2 and 3 (d) 1 and 3 only Answer: a) Answer: d) Explanation: Explanation: Coral Spawning is one of the annual Insolvency is the situation where the debtor is synchronised events to occur on the Great not in a position to pay back the creditor. Barrier Reef. For a corporate firm, the signs of this could It occurs by releasing tiny egg and sperm be a slow-down in sales, missing of payment bundles simultaneously from their gut cavity deadlines etc. into the water. Bankruptcy is the legal declaration of By expelling the eggs and sperm at the same Insolvency. So the former is a financial time, the coral increases the likelihood that condition and latter is a legal position. All fertilisation will take place. insolvencies need not lead to bankruptcy. The mass spawning occurs after a full moon The code will apply to all sort of business and only after rising water temperatures have entities including corporate companies, stimulated the maturation of the gametes within the adult coral. The day length, tide height and salinity levels also appear to be www.iasgatewayy.com

UPSC NOVEMBER- 2019 DAILY CURRENT AFFAIRS CLASS TEST - 18 factors in deciding when the event will 3. What is the correct sequence of occurrence happen. of the following pilgrimage sites as one The spawning lasts between a few days and a proceeds from East to West? week. This is because different species release 1. Kedarnath their eggs and sperm on different days to 2. Badrinath prevent hybrids from being produced. 3. Gangotri The phenomenon — which only happens at 4. Yamunotri night — resembles an underwater snowstorm. Select the correct answer using the code given But rather than being all white, there are also below clouds of red, yellow and orange. All the (a) 1-2-3-4 bundles rise slowly to the surface where the (b) 4-3-2-1 process of fertilisation begins. (c) 1-3-2-4 While spawning takes place on a large scale, (d) 2-1-3-4 it doesn’t happen across the entire Reef all at Answer: d) once.

Instead, the time of year that corals spawn 4. Which of the following statements is/are depends on their location. Those on inshore correct? reefs usually start spawning one to six nights 1. -II missile was developed by DRDO after the first full moon in October, whereas (Defence Research and Development those in outer reefs spawn during November Organization) or December. 2. It is a surface-to-surface medium range When an egg is fertilised by a sperm it nuclear capable missile. develops into coral larva called a planula that 3. It has a range of 2,000–3,500 km floats around in the water for several days or Select the correct answer using the codes weeks before settling on the ocean floor. After given below the planula has settled in a particular area it (a) 1 and 2 only starts to bud and the coral colony develops. (b) 1 and 3 only The mass spawning also provides ready food (c) 2 and 3 only for other marine creatures, particularly (d) 1, 2 and 3 nocturnal animals such as plankton and some Answer: d) fish species.

www.iasgatewayy.com

UPSC NOVEMBER- 2019 DAILY CURRENT AFFAIRS CLASS TEST - 18 5. Consider the following statements about 7. The Kalasa-Banduri drinking water project Ain-i-Akbari is supposed to be constructed in 1. It is a detailed document recording the (a) Andhra Pradesh personal life of Emperor Akbar (b) Karnataka 2. It is written by his court historian, Abu'l (c) Madhya Pradesh Fazl (d) Ladakh 3. It was written in the Persian language. Answer: b) Select the correct answer using the codes Explanation: given below On October 23, Environment, Forest and (a) 1 and 2 only Climate Change Minister Prakash Javadekar (b) 1 and 3 only announced on Twitter that the “Kalasa- (c) 2 and 3 only Banduri drinking water project in Karnataka (d) 1, 2 and 3 has been granted Environment Approval”. Answer: c) It was later deleted. Explanation: On Twitter, the Goa CM, who belongs to the The Ain-i-Akbari or the "Administration of BJP, posted: “[River] Mhadei [the water of Akbar", is a 16th-century detailed document whose tributaries, Kalasa and Banduri, recording the administration of the Mughal Karnataka wants to divert to the Malaprabha Empire under Emperor Akbar. basin in Karnataka] is more than mother to us . 6. Recently in news, SCALP and METEOR The Mhadei (or Mahadayi/Mahadeyi) rises in refers to, the Western Ghats in Khanapur taluk of Karnataka’s Belagavi district, and enters Goa (a) Special kind of Asteroids in the Sattari taluk of North Goa. Several (b) Advanced Radar systems in S-400 streams join it along its course, and the river (c) Missiles grows in volume to become the Mandovi, one (d) None of the above of Goa’s two major rivers, before flowing into Answer: c) the Arabian Sea at Panaji. Explanation: Ahead of handing over of the

first Rafale jet to India, European missile 8. Recently in news, the Ziz-Zag technology is maker MBDA has said that the aircraft with used in most advanced weapons package comprising (a) Automobiles for transition from BSIV to Meteor and Scalp missiles will provide the BSVI country unrivalled deep strike capability and (b) High power processors in Super air dominance in the region. Computers

(c) Long distane communication satellites

www.iasgatewayy.com

UPSC NOVEMBER- 2019 DAILY CURRENT AFFAIRS CLASS TEST - 18 (d) Brick Kiln to reduce ash content Rajyasabha, it is open to Loksabha to accept Answer: d) or to reject any or all of the commendations. Explanation: When the Loksabha chooses to accept or In new technology, 5 per cent ash is left after decline the money bill with or without the burning coal, while in old technology, 15 per recommendation, the money bill is deemed cent ash is generated. passed in both houses.

9. Consider the following Statements 1. All BRICS members are member of G-20. 2. BRICS was formed earlier than G-20. Which of the above statement(s) is/are Incorrect (a) 1 only (b) 2 only (c) Both 1 and 2 (d) Neither 1 nor 2 Answer: a) Explanation: BRICS as a concept evolved only as late as 2001.

10. What will follow if a Money Bill is substantially amended by the Rajya Sabha? (a) The Lok Sabha may still proceed with the Bill, accepting or not accepting the recommendations the Rajya Sabha (b) The Lok Sabha cannot consider the bill further (c) The Lok Sabha may send the Bill to the Rajya Sabha for reconsideration (d) The President may call a joint sitting for passing the Bill Answer: a) Explanation: When a money bill returns to the Loksabha with amendments made by the

www.iasgatewayy.com

UPSC NOVEMBER- 2019 DAILY CURRENT AFFAIRS CLASS TEST - 19 1. India is part of which the following Banks (c) Population of the world without mobile 1. Asian Infrastructure Investment Bank phones 2. New Development Bank (d) None of the above 3. European Bank for Reconstruction and Answer: a) Development (EBRD) Explanation: O3b Networks Ltd. is a (a) 1 only network communications service provider (b) 1 and 3 only building and operating a medium Earth orbit (c) 2 and 3 only (MEO) satellite constellation primarily (d) All the Above intended to provide voice and data Answer: d) communications to mobile operators and Explanation: Internet service providers. The name "O3b" The Asian Infrastructure Investment Bank stands for "Other 3 billion", referring to the (AIIB) is a multilateral development bank population of the world where broadband that aims to support the building of Internet is not currently available. infrastructure in the Asia-Pacific region. The bank currently has 74 members as well as 26 3. Satpura tiger resevers is present in prospective members from around the world. (a) Gujarat HQ: Beijing, China.India is a member of it, (b) Maharashtra with 2nd highest voting power (c) Madhya Pradesh The New Development Bank (NDB), formerly (d) Uttar Pradesh referred to as the BRICS Development Bank, Answer: c) is a multilateral development bank Explanation: Recently, the Satpura Tiger established by the BRICS states (Brazil, Reserve located in the Madhya Pradesh was Russia, India, China and South Africa) in news because of the presence of Mahua Headquarters: Shanghai tree in its buffer zone. The European Bank for Reconstruction and Development (EBRD) is an international 4. Za’ir-Al-Bahr (Roar of The Sea) is joint financial institution founded in 1991. exercise between Navies of India Headquarters: London (a) Saudi Arabia India is a financing member of EBRD (b) Qatar (c) Oman 2. "O3b - "Other 3 billion", recently in news (d) Iran stands for Answer: b) (a) Population of the world where broadband Explanation: Navies of India and Qatar Internet is not currently available have started a five-day bilateral maritime (b) Population of the world under poverty exercise at Doha that includes surface action,

www.iasgatewayy.com

UPSC NOVEMBER- 2019 DAILY CURRENT AFFAIRS CLASS TEST - 19 air defence, maritime surveillance as well as 7. Consider the following statements: social and sports events. 1. The city of Dhaka was developed along its banks as a prime trade centre. 5. Consider the following about United 2. The river was once admired by the Nations Population Fund (UNFPA) Mughals, for its strategic location for 1. It releases the State of World defence. Population-2019 3. At present, it is one of the most polluted 2. The goal of UNFPA is ensure rivers in the world. reproductive rights for all. The above statements refer to which of the Which of the above statements are correct? following rivers: (a) 1 only (a) Padma River (b) 2 only (b) Meghna River (c) Both 1 and 2 (c) Buriganga River (d) Neither 1 nor 2 (d) Teesta River Answer: c) Answer: c) Explanation: Factual Statements Explanation: The Buriganga flows past the southwest outskirts of Dhaka city, the capital 6. A deadlock between the Lok Sabha and the of Bangladesh. Rajya Sabha calls for a joint sitting of the It is one of the most polluted rivers in the Parliament during the passage of world. Some experts refer to it as the largest 1. Ordinary Legislation sewer in the world. Uncontrolled dumping of 2. Money Bill human and industrial waste and 3. Constitution Amendment Bill encroachment has made the river biologically Select the correct answer using the codes dead. given below: (a) 1 only 8. Consider the following statements (b) 2 and 3 only regarding the Maternity Benefit Scheme: (c) 1 and 3 only 1. Pradhan Mantri Matru Vandana Yojana (d) 1, 2 and 3 (PMMVY) is a maternity benefit Answer: a) programme being implemented in all Explanation: Factual statements about districts of the country. Joint Sitting 2. All Pregnant Women and Lactating Mothers (PW&LM), including employees of the Central Government or

www.iasgatewayy.com

UPSC NOVEMBER- 2019 DAILY CURRENT AFFAIRS CLASS TEST - 19 the State Governments or PSUs are also (JSY). Thus, on an average, a woman gets covered under the scheme. Rs. 6,000. 3. The eligible beneficiaries also receive • Implementation of the scheme is closely cash incentive of Rs. 6000 under Janani monitored by the central and state Suraksha Yojana (JSY). governments through the Pradhan Mantri Which of the statement(s) given above is/are Matru Vandana Yojana - Common correct? Application Software (PMMVY-CAS). (a) 1 only • PMMVY-CAS is a web-based software (b) 1 and 2 only application that enables tracking the (c) 1 and 3 only status of each beneficiary under the (d) 1, 2 and 3 scheme, resulting in expedited, Answer: c) accountable and better grievance Explanation: redressal. • Pradhan Mantri Matru Vandana Yojana • All Pregnant Women and Lactating (PMMVY) is a maternity benefit Mothers (PW&LM), excluding those who programme being implemented in all are in regular employment with the districts of the country with effect from 1st Central Government or the State January, 2017. Governments or PSUs or those who are in • It is a centrally sponsored scheme being receipt of similar benefits under any law executed by the Ministry of Women and for the time being in force. Child Development. • All eligible Pregnant Women and • Cash benefits are provided to pregnant Lactating Mothers who have their women in their bank account directly to pregnancy on or after 1st January 2017 for meet enhanced nutritional needs and the first child in the family. partially compensate for wage loss. • Beneficiaries receive a cash benefit of Rs. 9. In the context of the Deposit Insurance, 5,000 in three installments on fulfilling consider the following statements: the following conditions: 1. If a bank goes bust in India, a depositor 1. Early registration of pregnancy has claim to a maximum of Rs 1 lakh per 2. Ante-natal check-up account as insurance cover. 3. Registration of the birth of the child and 2. The cover of Rs 1 lakh per depositor is completion of first cycle of vaccination for provided by the Insurance Regulatory the first living child of the family. and development Authority (IRDA). • The eligible beneficiaries also receive cash 3. Cooperative Banks are excluded from incentive under Janani Suraksha Yojana the Deposit Insurance Cover.

www.iasgatewayy.com

UPSC NOVEMBER- 2019 DAILY CURRENT AFFAIRS CLASS TEST - 19 Which of the statement(s) is/are correct? (a) 1 only (a) 1 only (b) 2 only (b) 1 and 2 only (c) Both 1 and 2 (c) 3 only (d) Neither 1 nor 2 (d) 1 and 3 only Answer: d) Answer: a) Explanation: Explanation: • Jayakwadi Dam is located on the • Currently, in the event of a bank going Godavari River in Aurangabad district, bust in India, a depositor has claim to a Maharashtra. maximum of Rs 1 lakh per account as • The dam aims to address the dual insurance cover — even if the deposit in problems of flooding along the banks their account far exceeds Rs 1 lakh. This during monsoon months, and that of amount is termed ‘deposit insurance’. drought during the rest of the year, • Depositors holding more than Rs 1 lakh in particularly in the Marathwada region. their account have no legal remedy in case • The seismometer is an instrument used to of the collapse of the bank. measure and record seismic waves. • The cover of Rs 1 lakh per depositor is Seismic waves study helps geologists to provided by the Deposit Insurance and map the interior of the Earth, and Credit Guarantee Corporation (DICGC), a measure and locate earthquakes and other fully owned subsidiary of the Reserve ground motions like volcanic eruptions, Bank of India. explosions, etc. precisely. • The Rs 1 lakh-cover is for deposits in • A piezometer is an instrument used for commercial banks, regional rural banks measuring the pressure of a liquid or gas (RRBs), local area banks (LABs), and or things related to pressure (such as the cooperative banks also. compressibility of liquid). It measures the strain of a liquid or fuel. They are often 10. Consider the following places which were placed in boreholes to monitor the recently seen in news: pressure or depth of groundwater. 1. Jayakwadi Dam is located on the Periyar • A slope meter is used to measure the River in Ernakulam district, Kerala. angles of slope, elevation or despair of an 2. Willingdon Island is the largest artificial object. island in India, which forms part of the • Willingdon Island is a seaport located in Elephanta Island, in the state of the city of Kochi, Kerala. Maharashtra. • It is the largest man-made/artificial island Which of the following statement(s) given of India and is surrounded by backwaters above is/are correct?

www.iasgatewayy.com

UPSC NOVEMBER- 2019 DAILY CURRENT AFFAIRS CLASS TEST - 19 (a part of a river in which there is little or no current). • It was carved out of Vembanad Lake and is connected via road and rail. The island is connected to the mainland by the Venduruthy Bridge. • It is a major commercial centre and is home to the Kochi Naval Base of the Indian Navy, the Central Institute of Fisheries Technology and the Port of Kochi. • It was named after the Viceroy Lord Willingdon and was created artificially in 1936 during his rule to improve the trade relations of British India with the rest of the world.

www.iasgatewayy.com

UPSC NOVEMBER- 2019 DAILY CURRENT AFFAIRS CLASS TEST - 20 1. Consider the following Answer: d) communities/groups and their region Explanation: Article 371, Special Provisions 1. Beni Amer - Sudan of Maharashtra and Gujarat: 2. Uighurs - China Governor has “special responsibility” to 3. Sidama - Yemen establish “separate development boards” for 4. Houthi - Ethiopia “Vidarbha, Marathwada, and the rest of Which of the above are correctly matched? Maharashtra”, and Saurashtra and Kutch in (a) 3 and 4 only Gujarat; (b) 1 and 2 only It ensures “equitable allocation of funds for (c) 2 and 3 only developmental expenditure over the said (d) 1 and 4 only areas”, and “equitable arrangement providing Answer: b) adequate facilities for technical education and Explanation: 3 and 4 are interchanged vocational training, and adequate opportunities for employment” under the 2. Consider the following statements state government.

regarding Article 371. 3. Kimberly Process recently in news is related 1. It provides special provisions for the to: states of Maharashtra and Gujarat. (a) To remove conflict diamonds from the 2. Under this, The President has special global supply chain responsibilities to establish separate (b) Method to implement transparency in development boards for “Vidarbha, Electoral Bonds Marathwada, and the rest of (c) Initiative to curb illegal arms trade Maharashtra”, and “Saurashtra and (d) To decrease import of Gold to control Kutch in Gujarat”. Current Account Deficit 3. It ensures equitable arrangement Answer: a) providing adequate facilities for Explanation: Factual Statement technical education and vocational training, and adequate opportunities for 4. Consider the following regarding Nodal employment. Agencies for disaster management Disaster Which of the statement(s) given above is/are Nodal Agency correct? 1. Cyclones A. Indian Meteorological (a) 1 and 2 only. Directorate (b) 2 and 3 only. 2. Earthquakes B. Indian Meteorological (c) 1, 2 and 3. Directorate (d) 1 and 3 only.

www.iasgatewayy.com

UPSC NOVEMBER- 2019 DAILY CURRENT AFFAIRS CLASS TEST - 20 3. Floods C. Central Water Commission Explanation: National Company Law 4. Chemical Disasters D. Council of Appellate Tribunal (NCLAT) was constituted Scientific and Industrial Research (CSIR) under Section 410 of the Companies Act, Choose the correctly matched 2013 for hearing appeals against the orders of (a) 1 and 2 only National Company Law Tribunal(s) (NCLT), (b) 2 and 3 only with effect from 1st June, 2016.NCLAT is also (c) 1, 2 and 3 only the Appellate Tribunal for hearing appeals (d) All the above against the orders passed by NCLT(s) under Answer: c) Section 61 of the Insolvency and Bankruptcy Explanation: Nodal Agencies and disasters Code, 2016 (IBC), with effect from 1st associated December, 2016. Floods: CWC, Ministry of Water Resources NCLAT is also the Appellate Tribunal to hear Cyclones: Indian Meteorological Directorate and dispose of appeals against any direction Earthquakes: Indian Meteorological issued or decision made or order passed by Directorate the Competition Commission of India (CCI) Epidemics: Ministry of Health and Family Welfare 6. Consider the following statements Chemical Disasters: Ministry of regarding Electoral Bonds in India: Environment and Forests 1. Electoral bonds will allow donors to pay Industrial Disasters: Ministry of Labour political parties using banks as an Fire: Ministry of Home Affairs intermediary. 2. As per provisions of the Scheme, 5. The National Company Law Appellate electoral bonds may be purchased by a Tribunal (NCLAT) was established for citizen of India, or entities incorporated hearing appeals against the orders of or established in India. 1. National Company Law Tribunal(s) 3. Only the registered Political Parties (NCLT) which have secured not less than six per 2. Insolvency and Bankruptcy Board of cent of the votes polled in the last Lok India Sabha elections or the State Legislative 3. Competition Commission of India (CCI). Assembly are eligible to receive the Which of the above are correct? Electoral Bonds. (a) 1 and 3 only Which of the statement(s) given above is/are (b) 2 and 3 only correct? (c) 1 and 2 only (a) 1 only (d) All the above (b) 1 and 2 only Answer: d) www.iasgatewayy.com

UPSC NOVEMBER- 2019 DAILY CURRENT AFFAIRS CLASS TEST - 20 (c) 2 and 3 only 7. The Suez Canal connects which of the (d) 1 and 3 only following seas? Answer: b) (a) Red Sea and Caspian Sea Explanation: (b) Caspian Sea and Mediterranean Sea Electoral bonds will allow donors to pay (c) Mediterranean Sea and Red Sea political parties using banks as an (d) Red Sea and Arabian Sea intermediary. Answer: c) Although called a bond, the banking Explanation: instrument resembling promissory notes will Suez Canal is an artificial sea-level waterway not carry any interest. in Egypt. It links the Mediterranean with the The electoral bond, which will be a bearer Red Sea through Isthmus of Suez. It was first instrument, will not carry the name of the opened in 1869 after a decade of payee and can be bought for any value, in construction. multiples of Rs 1,000, Rs 10,000, Rs 1 lakh, Strategically and economically it is one of the Rs 10 lakh or Rs 1 crore. most important waterways in the world As per provisions of the Scheme, electoral providing the shortest sea link between Asia bonds may be purchased by a citizen of India, and Europe. or entities incorporated or established in Its opening had provided alternative route to India. sea voyage between Europe and India. It had A person being an individual can buy reduced the sea voyage distance between electoral bonds, either singly or jointly with these two regions by about 7,000 kilometres other individuals. without navigating around Africa. Only the registered Political Parties which The canal was nationalized by Egypt in 1956 have secured not less than one per cent of the after brief war against the UK, France and votes polled in the last Lok Sabha elections or Israel. The canal has been a significant the State Legislative Assembly are eligible to income source for Egypt since then. receive the Electoral Bonds. The electoral bonds are aimed at rooting out 8. With regards to Contempt of Court, the current system of largely anonymous cash consider the following statements: donations made to political parties which 1. Civil contempt is a contempt is the lead to the generation of black money in the publication whether by words, spoken economy. or written, or by signs, or by visible representation. 2. Criminal contempt is a ‘willful disobedience to any judgment, decree, direction, order, writ or other processes

www.iasgatewayy.com

UPSC NOVEMBER- 2019 DAILY CURRENT AFFAIRS CLASS TEST - 20 of a Court or willful breach of an 9. Which one of the following is the best undertaking given to the court’. description of the term 'ecosystem'? 3. The Contempt of Courts Act, 1971, very (a) A community of organisms interacting clearly states that fair criticism of any with one another. case which has been heard and decided (b) That part of the Earth which is inhabited is not contempt. by living organisms. Which of the statements given above are (c) A community of organisms together with incorrect? the environment in which they live (a) 1 and 2 only (d) The flora and fauna of a geographical area (b) 2 and 3 only Answer: a) (c) 1 and 3 only Explanation: (d) 1, 2 and 3 An ecosystem includes all of the living things Answer: a) (plants, animals and organisms) in a given Explanation: area, interacting with each other, and also In India, the Contempt of Courts Act, 1971, with their non-living environments (weather, divides contempt into civil contempt and earth, sun, soil, climate, atmosphere). criminal contempt. ‘Civil contempt’ is a ‘willful disobedience to 10. INS Trikand which was recently seen in any judgment, decree, direction, order, writ news is a or other processes of a Court or willful breach (a) Submarine of an undertaking given to the court’. (b) Light aircraft carrier ‘Criminal contempt’ is ‘the publication (c) Frigate (whether by words, spoken or written, or by (d) Anti submarine warfare signs, or by visible representation, or Answer: c) otherwise) of any matter or the doing of any Explanation: other act whatsoever which: Trikand belongs to the Talwar class of guided 1. Scandalises or tends to scandalise, or missile frigates. These are modified Krivak lowers or tends to lower the authority of, III-class frigates built by Russia. any court. These ships use stealth technologies and a 2. Prejudices, or interferes or tends to special hull design to ensure a reduced radar interfere with the due course of any cross section. judicial proceeding. It was recently used in Bilateral Maritime 3. Interferes or tends to interfere with, or Exercise Zair-Al-Bahr (Roar of the Sea). obstructs or tends to obstruct, the administration of justice in any other

manner.’ www.iasgatewayy.com

UPSC NOVEMBER- 2019 DAILY CURRENT AFFAIRS CLASS TEST - 21 1. Consider the following statements June to December, with the peak period regarding Olive Ridleys: in September and October. 1. The Olive ridley turtles are the most • Gahirmatha beach in Odisha’s abundant of all sea turtles found in the Kendrapara district is known as world’s world, inhabiting warm waters of the largest rookery of olive ridley species. Atlantic and Indian oceans only. • Nasi II Island is part of Gahirmatha 2. The species is listed as Vulnerable in the Sanctuary in Odisha, one of the world's IUCN Red List and Schedule 1 in largest nesting grounds of olive ridley Wildlife Protection Act, 1972. turtles. 3. In India they are constrained to the • Olive-ridleys face serious threats across eastern coasts of Rushikulya River and their migratory route, habitat and nesting beaches, due to human activities such as Gahirmatha coast of Odisha. unfriendly turtle fishing practices, Which of the statement(s) given above is/are development, and exploitation of nesting correct? beaches for ports, and tourist centers. (a) 2 only • They are not constrained to eastern (b) 1 and 2 only coasts; they are also found in the coast of (c) 3 only Maharashtra. (d) 1, 2 and 3

Answer: a) 2. In the context of Bharat Stage Norms, Explanation: consider the following statements: • The Olive ridley turtles are the second 1. The Central Pollution Control Board smallest and most abundant of all sea (CPCB) is responsible for deciding the turtles found in the world, inhabiting fuel standard in the country. warm waters of the Pacific, Atlantic and 2. Government of India has decided to roll Indian oceans. on from BS-V norms to BS-VI norms • They are best known for their unique from April 2020. mass nesting called Arribada, where 3. On-board diagnostics (OBD) is thousands of females come together on mandatory for all BS-VI automobiles the same beach to lay eggs. which is a sophisticated emission • The species is listed as Vulnerable in the control device for optimum efficiency IUCN Red List, Appendix 1 in CITES, and throughout the life of the vehicle. Schedule 1 in Wildlife Protection Act, Which of the statement given above is 1972. correct? • They nests twice/thrice in a year. Its (a) 1 only nesting season ranges worldwide from (b) 2 only

www.iasgatewayy.com

UPSC NOVEMBER- 2019 DAILY CURRENT AFFAIRS CLASS TEST - 21 (c) 3 only (c) Providing reservation for backward (d) None Classes. Answer: c) (d) Reservation for Women in Panchayats up Explanation: to 33% and reservation of Seats for • The Bharat Stage (BS) is emission SC/ST, in Panchayats, in proportion to standard instituted by the Government of their population. India to regulate the output of air Answer: c) pollutants from motor vehicles. Explanation: • The Environment Ministry is responsible Compulsory Provisions Include: for deciding the fuel standard in the • Organisation of Gram Sabha; country. The Central Pollution Control • Creation of a three-tier Panchayati Raj Board (CPCB) implements these Structure at the district, block and Village standards. levels; • To curb growing menace of air pollution • All the Seats in a Panchayat shall be filled through the vehicles emission, the by persons Chosen by direct elections Government of India has decided to from territorial Constituencies in the leapfrog from the exiting BS – IV norms Panchayat area; to the BS- VI, thereby skipping the BS – V • The minimum age for contesting elections norms, and to implement the BS – VI to Panchayats to be 21 years. norms with effect from 1st April 2020. • Reservation for Women in Panchayats up • The shift makes on-board diagnostics to 33% (OBD) mandatory for all automobiles. The • Reservation of Seats for SC/ST, in OBD unit will be able to identify likely Panchayats, in proportion to their areas of malfunction by means of fault population; codes stored on a computer ensuring that • Creation of a State Election Commission sophisticated emission control device to conduct elections. which is fitted in a BS-VI vehicle runs at • Fixed 5 years tenure of Panchayats. optimum efficiency throughout the life of • Each State is to Constitute a State Finance the vehicle. Commission every five years to review the financial position of the Panchayat. 3. Which of the following is not a compulsory Voluntary Provisions include: feature of Panchayati Raj? • Giving Voting rights to members of the (a) Organisation of Gram Sabha Union and State Legislatures in these (b) Creation of a three-tier Panchayati Raj bodies; Structure at the district, block and Village • Providing reservation for backward levels Classes. www.iasgatewayy.com

UPSC NOVEMBER- 2019 DAILY CURRENT AFFAIRS CLASS TEST - 21 • Giving the Panchayats financial autonomy Santushti Scheme (Public Private Partnership and thereunder the power to levy taxes, for sterilization services) fees, etc. National Helpline (for information on family • Devolution of Powers to the Panchayat planning). bodies to perform functions as provided in the XI Schedule. 5. Consider the following statements 1. The International Court of Justice is the 4. With respect to Jansankhya Sthirata Kosh principal judicial organ of the United (JSK) an autonomous body under Ministry Nations of Health and Family Welfare, consider the 2. The International Criminal Court is an following schemes: intergovernmental organization and 1. Prerna Scheme international tribunal that sits in The 2. Santushti Scheme Hague in the Netherlands. 3. National Helpline for information on 3. India is a member of Permanent Court family planning of Arbitration. Which of the above schemes are implemented Which of the statement(s) given above is correct? by it: (a) 1 only (a) 1 only (b) 1 and 3 only (b) 1 and 3 only (c) 2 and 3 only (c) 2 and 3 only (d) All the above (d) All the above Answer: d) Answer: d) Explanation: Factual Statement Explanation: Jansankhya Sthirata Kosh

(JSK) is a registered society of the Ministry of 6. Recently in news, Europa is moon of Health and Family Welfare started with a Rs (a) Jupiter 100 crore grant from government. (b) Mars The society was established with the objective (c) Uranus of highlighting the need for population (d) Neptune stabilization. Its accounts can be audited by Answer: a) the CAG. Explanation: NASA’s has confirmed the Jansankhya Sthirata Kosh (JSK) an presence water vapour for the first time above autonomous body under Ministry of Health the surface of Jupiter’s Moon Europa. and Family Welfare, implemented the

following schemes:

Prerna Scheme (for delaying marriage,

childbirth and spacing),

www.iasgatewayy.com

UPSC NOVEMBER- 2019 DAILY CURRENT AFFAIRS CLASS TEST - 21 7. Consider the following pairs: • It has been originated from Tibet and 1. Garba : Gujarat popularly practice in India, Nepal, 2. Mohiniattam : Odisha Bhutan, Mongolia and Russia. 3. Yakshagana : Karnataka • The majority of theory and practice of Which of the pairs given above is / are Sowa-Rigpa is similar to “Ayurveda”. incorrectly matched? • The impact of Sowa-Rigpa along with (a) 1 only Buddhism and other Tibetan art and (b) 2 only sciences were spread in neighboring (c) 1 and 3 only Himalayan regions. (d) 1, 2 and 3 • In India, this system is widely practice in Answer: b) Sikkim, Arunachal Pradesh, Darjeeling Explanation: Mohiniattam: Kerala (West Bengal), Dharamsala, Lahaul and Spiti (Himachal Pradesh) and Ladakh 8. Consider the following statements region of Jammu & Kashmir. regarding ‘Sowa-Rigpa’ 1. It is a discipline that is practiced 9. Consider the following statements especially for self-defense, exercise, and regarding ‘Organoids’ spiritual growth. 1. Organoids are a group of cells grown in 2. It is also called wushu or quanfa. laboratories into three-dimensional, 3. It originated from Mongolia and miniature structures that mimic the cell popularly practiced in India, Nepal, arrangement of a fully-grown organ. Bhutan, Mongolia and Russia. 2. Organoids are grown in the lab using Which of the statement(s) given above is/are stem cells that can become any of the correct? specialised cells seen in the human (a) 1 only body, or from induced pluripotent stem (b) 1 and 2 only cells (c) 1, 2 and 3 3. Organoids of the brain, small intestine, (d) None of the above kidney, heart, stomach, eyes, liver, Answer: d) pancreas, prostate, salivary glands, and Explanation: inner ear have been successfully • “Sowa-Rigpa” commonly known as developed in the laboratory. Tibetan system of medicine is one of the Which of the statement(s) given above is/are oldest, Living and well documented correct? medical tradition of the world. (a) 1 only (b) 1 and 2 only

www.iasgatewayy.com

UPSC NOVEMBER- 2019 DAILY CURRENT AFFAIRS CLASS TEST - 21 (c) 1, 2 and 3 10. Which of the following statements is/are (d) None of the above incorrect? Answer: c) (a) Plague is also known as the "Black Death". Explanation: (b) Plague is caused by the bacteria Yersinia • Organoids are a group of cells grown in pestis, a zoonotic bacterium usually found laboratories into three-dimensional, in small mammals and their fleas. miniature structures that mimic the cell (c) There is no anti-biotic treatment to treat arrangement of a fully-grown organ. plague • They are tiny organ-like structures that do (d) None of the above not achieve all the functional maturity of Answer: c) human organs but often resemble the Explanation: early stages of a developing tissue. • Plague is caused by the bacteria Yersinia • Organoids are grown in the lab using stem pestis, a zoonotic bacterium usually found cells that can become any of the in small mammals and their fleas. specialised cells seen in the human body, • People infected with Y. pestis often or from induced pluripotent stem cells develop symptoms after an incubation (iPSC). period of one to seven days. • Stem cells are provided with nutrients and • There are two main clinical forms of other specific molecules to grow and plague infection: bubonic and pneumonic. become cells resembling a specific organ. Bubonic plague is the most common form • The growing cells are capable of self- and is characterized by painful swollen organizing into cellular structures of a lymph nodes or 'buboes'. specific organ and can partly replicate • Plague is transmitted between animals complex functions of mature organs like and humans by the bite of infected fleas, physiological processes of regeneration. direct contact with infected tissues, and • Organoids of the brain, small intestine, inhalation of infected respiratory kidney, heart, stomach, eyes, liver, droplets. pancreas, prostate, salivary glands, and • Antibiotic treatment is effective against inner ear have already been developed in plague bacteria, so early diagnosis and the laboratory. early treatment can save lives.

www.iasgatewayy.com

UPSC NOVEMBER- 2019 DAILY CURRENT AFFAIRS CLASS TEST - 22 1. Consider the following statements Short Notice Questions: Such questions can regarding Parliamentary proceedings be asked orally in the House after the 1. Short Notice Questions can be asked Question Hour or as the first item in the orally in the House after the Question agenda where there is no Question Hour at a Hour or as the first item in the agenda notice shorter than that prescribed for where there is no Question Hour. Starred and Unstarred Questions. These must 2. Starred Questions must be related to a relate to a subject-matter considered by the Chairman to be of urgent public importance. subject-matter considered by the

Chairman to be of urgent public 2. Biological Oxygen Demand (BOD) is a importance. standard criterion for 3. Unstarred Questions are those which (a) Measuring oxygen levels in blood answers are desired to be given orally on (b) Computing oxygen levels in forest the floor of the House during the ecosystems Question Hour (c) Pollution assay in aquatic ecosystems Which of the statement(s) given above is/are (d) Assessing oxygen levels in high altitude incorrect? regions. (a) 1 and 2 only Answer: c) (b) 2 and 3 only Explanation: (c) 1, 2 and 3 only Water pollution by organic wastes is (d) 1 and 3 only measured in terms of Biochemical Oxygen Answer: b) Demand (BOD). Explanation: BOD is the amount of dissolved oxygen Starred Questions: These are Questions to needed by bacteria in decomposing the which answers are desired to be given orally organic wastes present in water. It is on the floor of the House during the Question expressed in milligrams of oxygen per litre of Hour. These are distinguished in the printed water. lists by asterisks. 15 such questions are listed The higher value of BOD indicates low each day. Dissolved Oxygen content of water. Unstarred Questions: These are Questions to Since BOD is limited to biodegradable which written answers are given by Ministers materials, it is not a reliable method of which are deemed to have been laid on the measuring water pollution. Table of the House at the end of the Question Chemical oxygen demand (COD) is a slightly Hour. Upto 160 such questions are listed each better mode used to measure pollution load day in a separate list. in the water.

www.iasgatewayy.com

UPSC NOVEMBER- 2019 DAILY CURRENT AFFAIRS CLASS TEST - 22 COD measures the amount of oxygen in parts 4. The Dumbur Hydro Electric Plant is per million required to oxidise organic commissioned in (biodegradable and non-biodegradable) and (a) Tripura oxidizable inorganic compounds in the water (b) Meghalaya sample. (c) Manpur (d) Nagaland 3. Consider the following statements Answer: a) 1. Diastrophism is an endogenic Explanation: Location based question geomorphic process.

2. Plate tectonics involving horizontal 5. In relation to ODF+ and ODF++ protocol, movements of crustal plates are form of consider the following statements: diastrophism. 1. A city /ward is notified as ODF+ 3. Epeirogenic processes are also a form of city/ward if, at any point of the day, not diastrophism. a single person is found defecating in Which of the statement(s) given above is/are the open. correct? 2. A city /ward is notified as ODF++ (a) 1 only city/ward if, at any point of the day, not (b) 2 and 3 only a single person is found defecating and (c) 3 only /or urinating in the open, and all (d) 1, 2 and 3. community ad public toilets are Answer: d) functional and well maintained" Explanation: Which of the statement(s) given above is/are Diastrophism and volcanism are endogenic incorrect? geomorphic processes. (a) 1 only Weathering, mass wasting, erosion and (b) 2 only deposition are exogenic geomorphic (c) Both 1 and 2 processes. (d) Neither 1 and 2 Diastrophism is the general term applied to Answer: c) slow bending, folding, warping and Explanation: fracturing. To ensure the sustainability and long-term Processes like orogeny, epeirogeny and plate impacts, Open Defecation+ and Open tectonics are forms of diastrophism. Defecation++ are used.

In general, ODF protocol "A city /ward is

notified as ODF city/ward if, at any point of

www.iasgatewayy.com

UPSC NOVEMBER- 2019 DAILY CURRENT AFFAIRS CLASS TEST - 22 the day, not a single person is found 3. Maharashtra and Karnataka currently defecating in the open" have no designated wetlands under The ODF+ protocol says that a city, ward or Ramsar circle could be declared ODF+," at any point Which of the statement(s) given above is/are of the day, not a single person is found correct? defecating and /or urinating in the open, and (a) 1 only all community ad public toilets are functional (b) 2 and 3 only and well maintained" (c) 3 only In ODF++ protocol adds the condition that (d) 1 and 3 only. "faecal sludge/septage and sewage is safety Answer: d) managed and treated, with no discharging or Explanation: dumping of untreated faecal sludge/septage Wetlands (Conservation and Management) and sewage in drains, water bodies or open Rules, 2017 rules shall not apply to the area" wetlands falling in areas covered under the Indian Forest Act, 1927, the Wildlife 6. Recently in news Idris Elba named after an (Protection) Act, 1972, the Forest actor is a species of (Conservation) Act, 1980, the State Forest (a) Spider Acts, and the Coastal Regulation Zone (b) Wasp Notification, 2011. (c) Frog (d) Snake 8. Consider the following statements about Answer: b) UNEP Explanation: A new wasp spices was named 1. The United Nations Environmental after British actor Idris Elba. It is found living Programme (UNEP) was founded in as a parasite in eggs of another insect, known June 1972 as a result of the Stockholm as the Bagrada bug, which is a major pest of Conference on the Human cruciferous vegetables. Environment.

2. IPCC was created in 1988 by the World 7. Consider the following statements Meteorological Organization (WMO) 1. Sambar Lake is a Ramsar Wetland Site and the United Nations Environment in Rajasthan Programme (UNEP). 2. Wetlands (Conservation and 3. The Production Gap Report 2019 which Management) Rules, 2017 rules apply to talks about the gap between the targets the wetlands falling in areas covered of the Paris Agreement and countries under the Indian Forest Act, 1927

www.iasgatewayy.com

UPSC NOVEMBER- 2019 DAILY CURRENT AFFAIRS CLASS TEST - 22 dependent on fossil fuels is a A descendant of the Bodhi tree under which publication of UNEP. the Buddha is said to have sat until he Which of the statement(s) given above is/are attained enlightenment stands adjacent to the correct? temple. (a) 1 only Ashoka’s stone slab purporting to mark the (b) 2 and 3 only exact position where the Buddha sat is (c) 3 only traditionally called the Buddha’s vajrasana (d) All the above (literally “diamond throne” or “thunder Answer: d) seat”). Explanation: Factual Statements It was recognized as a UNESCO World Heritage site in 2002. 9. ‘Maha Bodhi Temple Complex’, recently seen in news was initially built by 10. The recently proposed Industrial Relations (a) Ashoka Code Bill, 2019 amalgamates which of the (b) Kanishka following acts? (c) Adi Shankara 1. The Trade Unions Act, 1926 (d) Dharmapala 2. The Industrial Employment (Standing Answer: a) Orders) Act, 1946 Explanation: 3. The Industrial Disputes Act, 1947 The Maha Bodhi Temple Complex is one of Select the correct answer using the codes the four holy sites related to the life of the given below: Lord Buddha, and particularly to the (a) 1 and 2 only attainment of Enlightenment (Bodhi). (b) 2 and 3 only The other three are: Lumbini (Birth) in (c) 1 and 3 only Nepal, Sarnath (Dharma-Chakra-Pravartana (d) 1, 2 and 3 - 1st Sermon) and Kushinagar Answer: d) (Mahaparinirvana- death) in Uttar Pradesh. Explanation: The original structure was built by the The Industrial Relations Code Bill, 2019 Mauryan emperor Ashoka. proposes to amalgamate The Trade Unions However, it was reconstructed entirely in Act, 1926, The Industrial Employment brick in late Gupta period. The present (Standing Orders) Act, 1946, and The temple dates from the 5th or 6th centuries. Industrial Disputes Act, 1947. Apart from The site of the Maha Bodhi Temple provides offering some degree of flexibility on exceptional records of the events associated government permissions for retrenchment, with the life of Buddha and subsequent the most important aspect of the Bill is that it worship. presents the legal framework for ushering in

www.iasgatewayy.com

UPSC NOVEMBER- 2019 DAILY CURRENT AFFAIRS CLASS TEST - 22 the concept of ‘fixed-term employment’ through contract workers on a pan-India basis. Currently, companies hire contract workers through contractors. With the introduction of fixed-term employment, they will be able to hire workers directly under a fixed-term contract, with the flexibility to tweak the length of the contract based on the seasonality of industry. These workers will be treated on a par with regular workers during the tenure of the contract. The move to include it in a central law will help in wider reach, and states are expected to follow similar applicability. The government had tried a move last year to apply fixed-term employment across “central sphere establishments” (which are establishments under the authority of the central government, Railways, mines, oilfields, major ports, or any other central public sector undertaking) in all sectors, but it failed to elicit the desired results as states did not notify similar provisions for it. The Bill now ensures a pan-India impact of this move.

www.iasgatewayy.com

UPSC NOVEMBER- 2019 DAILY CURRENT AFFAIRS CLASS TEST - 23 1. ‘Ghoramara Island’ sometimes seen in news Which of the statement(s) given above is/are belongs to which country? correct? (a) India (a) 1 only (b) Bangladesh (b) 1 and 3 only (c) Myanmar (c) 1, 3 and 4 only (d) Maldives (d) All the above Answer: a) Answer: b) Explanation: Explanation: Ghoramara Island in the Ganga estuary of GDP is essentially about where the West Bengal is slowly being submerged by production takes place whereas GNP is about rising sea levels, forcing people to migrate in who produces. large numbers. If it is an open economy then GNP may or Neighbouring islands of Sagar and Mousuni may not greater than GDP. If it is a closed are also under threat of submergence. economy then GDP will always be equal to The story of Ghoramara shows how climate GNP. change is changing the way people live — how GDP is better metric of measure than GNP. it divides families, breaks social taboos and hastens forced migration. 3. ‘Hong Kong Convention’ recently seen in The largely poor people in the island (45 news is related to percent live below the poverty line) are under (a) Safe and Environmentally Sound enormous socioeconomic stress that has Recycling of Ships upturned their lives. (b) Environmental Health Hazards (c) Smart Cities Programme 2. Consider the following statements with (d) Prevention on hunting of Whales respect to GDP and GNP: Answer: a) 1. GDP and GNP, both measure the Explanation: market value of all goods and services The Hong Kong International Convention for produced for final sale in an economy. the Safe and Environmentally Sound 2. If it is an open economy then GNP will Recycling of Ships, 2009 (the Hong Kong always be larger than GDP. Convention), was adopted at a diplomatic conference held in Hong Kong, China in 3. If it is a closed economy then GDP will 2009. always be equal to GNP. It was adopted by the International Maritime 4. GNP is better metric of measure for the Organization (IMO) in 2009. overall economic condition of a country The Convention is aimed at ensuring that than GDP. ships, when being recycled after reaching the

www.iasgatewayy.com

UPSC NOVEMBER- 2019 DAILY CURRENT AFFAIRS CLASS TEST - 23 end of their operational lives; do not pose any from 3.3% and initiated a slew of measures unnecessary risks to human health, safety that are being dubbed as mini-budget. and to the environment. The RBI had formed a committee chaired by It also addresses concerns raised about the former Governor Bimal Jalan to review its working and environmental conditions at economic capital framework and suggest the many of the world’s ship recycling locations. quantum of excess provision to be transferred The Convention is yet to come into force to the government. because it has not been ratified by 15 nations, The panel recommended a clear distinction representing 40 per cent of the world between the two components of the economic merchant shipping by gross tonnage capital of RBI i.e. Realized equity and (capacity) and a maximum annual ship Revaluation balances. recycling volume of not less than 3 per cent of Revaluation reserves comprise of periodic the combined tonnage of the countries. marked-to-market unrealized/notional gains/losses in values of foreign currencies 4. Transfer of RBI Surplus to the Central and gold, foreign securities and rupee Government was done recently under the securities, and a contingency fund. recommendation of Realized equity, which is a form of a (a) Rangarajan Committee contingency fund for meeting all risks/losses (b) Bimal Jalan Committee primarily built up from retained earnings. It (c) Narashimman Committee is also called the Contingent Risk Buffer (d) Tapan Ray Committee (CBR). Answer: b) The Surplus Distribution Policy of RBI that Explanation: was finalized is in line with the Recently, the Reserve Bank of India (RBI) has recommendations of the Bimal Jalan decided to transfer Rs 1.76 lakh crore to the committee. Central government, which may help the The Jalan committee has given a range of 5.5- government in dealing with the economic 6.5% of RBI's balance sheet for Contingent slowdown. Risk Buffer. The Rs 1.76 lakh crore includes the central Adhering to the recommendations, the RBI bank’s 2018-19 surplus of ₹1.23 lakh crore has decided to set the CBR level at 5.5% of the and Rs 52,637 crore of excess provisions balance sheet, while transferring the identified as per the revised Economic Capital remaining excess reserves worth ₹52,637 Framework (recommended by Bimal Jalan crore to the government. Committee). If CBR is below the lower bound of The government already had revised requirement, risk provisioning will be made downward the fiscal deficit target to 3.4% to the extent necessary and only the residual

www.iasgatewayy.com

UPSC NOVEMBER- 2019 DAILY CURRENT AFFAIRS CLASS TEST - 23 net income (if any) transferred to the 6. Consider the following statements Government. regarding ‘Swachchta Hi Seva Campaign’ However, keeping CBR at a lower range of 1. It is acountrywide awareness and 5.5% will reduce RBI's space to manoeuvre mobilization campaign. monetary policy. 2. The campaign is coordinated by the Ministry of Jal Shakthi. 5. Recently, India became the last major 3. The ‘Swachchta Hi Seva- 2019’ cricketing nation to enter the pink-ball Test Campaign will be focused on Plastic era. The ‘Pink Ball’ is used because Waste Management. (a) It represents India’s solidarity to the Which of the statement(s) given above is/are women, in the field of cricketing. incorrect? (b) It symbolizes the commitment of India (a) 2 only towards the Sustainable Development (b) 3 only Goals (SDG). (c) 1and 3 only (c) It is better visible in the artificial lights of (d) None of the above the stadium once the natural light fades Answer: d) away. Explanation: (d) BCCI favoured using of Pink Ball to Swachhata Hi Seva (SHS) has been symbolize the change in its sponsorship. celebrated annually since 2017 inthe run up Answer: c) to Gandhi Jayanti. Explanation: The campaign has mobilized crores of people Why Pink? - Pink was the consensus colour every year to create awareness forSwachhata after ball makers tried optic yellow and bright 2019 campaign will be focused on Plastic orange, which were easy to spot on the grass, Waste Management, given the detrimental and by fielders taking high catches. (making effect of uncollected plastic waste on human it better visible in the artificial lights) beings, animals and the environment at large. Are Pink balls different? - all cricket balls are 7. Consider the following statements made of cork, rubber and woollen yarn, using 1. Ashvaghoshalived in the court similar production techniques. The colour of ofKanishka, the most famous Kushana the dye on the tanned cowhide, and the difference in ‘finishing’ decide in which ruler. format a ball is used. 2. Ashvaghoshacomposedthe Buddhacharita, a biography of the Buddha. Which of the statement(s) given above is/are correct?

www.iasgatewayy.com

UPSC NOVEMBER- 2019 DAILY CURRENT AFFAIRS CLASS TEST - 23 (a) 1 only All the Union and State Government (b) 2 only departments including the Indian Railways, (c) Both 1 and 2 Defence and Posts and Telecommunications. (d) None of the above About 1500 public commercial enterprises Answer: c controlled by the Union and State Explanation: governments, i.e. government companies and The fourth Buddhist council was held in corporations. 72 A.D at Kundalvana, Kashmir. Around 400 non-commercial autonomous It was presided by Vasumitra, while bodies and authorities owned or controlled by Asvaghosa was his deputy. the Union or the States. The council was held under the patronage of Bodies and authorities substantially financed Kushan King Kanishka of Kushan Empire. from Union some of the local bodies and Here, the Buddhism was divided into two Panchayati Raj Institutions which are critical sects namely Mahayana and Hinayana. grass root agencies for implementation of Ashvaghosha composed the Buddhacharita, a developmental programmes and delivery of biography of the Buddha. services.

9. Consider the following statements 8. Which of the following organisations are regarding ‘Nicotine’ subjected to the audit of the CAG? 1. Use of nicotine as an ingredient in any 1. All the Union and State Government food item is allowed in India, under the departments. Food Safety and Standards Act, 2006. 2. Non-commercial autonomous bodies 2. Nicotine and Nicotine Sulphate are and authorities owned or controlled by listed as hazardous chemicals under the the Union or the States. Environment (Protection) Act, 1986. 3. Private entities regulated by the Union 3. Nicotine is Listed as an insecticide in and State governments. the Schedule of insecticides under the Which of the given above is/are correct? Insecticide Act 1968, and subsequently (a) 1 only its use as a pesticide is banned by (b) 1 and 3 only (c) 1 and 2 only Government of lndia. (d) 1, 2 and 3 Which of the statement(s) given above is/are Answer: c) correct? Explanation: (a) 1 only The organisations subject to the audit of the (b) 2 only CAG are: (c) 2 and 3 only (d) 1, 2 and 3

www.iasgatewayy.com

UPSC NOVEMBER- 2019 DAILY CURRENT AFFAIRS CLASS TEST - 23 Answer: b) Explanation: Explanation: Mahadayi river rises in the Western Ghats, Nicotine is prohibited for use as an ingredient from the Bhimgad Wildlife Sanctuary in in any food item under the Food Safety and Khanapur taluk of Karnataka’s Belagavi Standards (Prohibition and Restrictions on district. Flowing westward, it enters Goa from Sales) Regulation, 2011 of the Food Safety Sattari taluk of North Goa districts and Standards Act, 2006. River Mandovi, also known as Mahadayi or Nicotine and Nicotine Sulphate are listed as Mhadei river is regarded as the lifeline of hazardous chemicals in the Manufacture, Goa. The river originates from a cluster of 30 Storage and Import of Hazardous Chemical springs at Bhimgad in the Western Ghats in Rules, 1989 made under theEnvironment the Belagavi district of Karnataka and has a (Protection) Act, 1986. total 2,032 km2 catchment area of which Nicotine is Iisted as an insecticide in the 1,580 km2, 375 km2 and 77 km2 catchment Schedule of lnsecticides under the Insecticide area are in Goa, Karnataka and Maharashtra Act 1968, and subsequently its use as a respectively. pesticide is also highly restricted The Kalasa-Banduri Nala is a project byGovernment of lndia. undertaken by the Government of Karnataka to improve drinking water supply to the 10. With reference to River Mandovi, Consider Districts of Belagavi, Dharwad and Gadag. It the following statements: involves building across Kalasa and Banduri, 1. It rises in the Western Ghats, from the two tributaries of the Mahadayi river to divert Bhimgad Wildlife Sanctuary in 7.56 TMC of water to the Malaprabha river, Karnataka. which supplies the drinking water needs of 2. It flows through Karnataka and the said 3 districts, i.e., Dharwad, Belagavi Maharashtra only. and Gadag.

3. Government of India has approved

Kalasa Banduri Nala project to improve drinking water supply. Which of the statement(s) given above is/are correct? (a) 1 only (b) 1 and 2 only (c) 1 and 3 only (d) 2 and 3 only Answer: a)

www.iasgatewayy.com

UPSC NOVEMBER- 2019 DAILY CURRENT AFFAIRS CLASS TEST - 25 3. Which of the following is/are true about ‘Sri 1. Dokdo islands sometimes seen in news Nityananda Prabhu’? located in? 1. He played a significant role in the (a) Sea of Japan foundation and development of the (b) South china Sea (c) Yellow Sea Vaishnava movement in Tamil Nadu. (d) East Sea 2. He was a disciple of Chaitanya Answer: a) Mahaprabhu. Explanation: 3. He denounced the Caste System and It is located in Sea of Japan and dispute helped in the revival of Hinduism in between Japan and South Korea. Eastern India. They are 210 kilometers across the water (a) 1 and 2 only from Japan and South Korea and are only 19 (b) 2 and 3 only hectares in area. (c) 3 only These islands are called the Liancourt Rocks (d) 1, 2 and 3 in the west, Takeshima in Japan and Dokdo Answer: b) in South Korea. Explanation: ‘Sri Nityananda Prabhu’ is a contemporary 2. In the context to Ken Betwa interlinking and chief disciple of Chaitanya Mahaprabhu. project, consider the following statements: Nityananda Prabhu played a significant role 1. The proposed project will be located in in the foundation and development of the Panna tiger reserve. Vaishnava movement in West Bengal. 2. Both the rivers are tributaries of Ganga. Chaitanya Mahaprabhu and Sri Nityananda Which of the statement(s) given above is/are Prabhu are credited with the revival of correct? Hinduism in Eastern India, plagued mainly (a) 1 only by the caste system, which they denounced. (b) 2 only Major Vaishnava literature of medieval (c) Both 1 and 2 period, came from them or their disciples.

(d) Neither 1 and 2 4. Consider the following statements Answer: a) regarding the Observers of ECI Explanation: 1. They are appointed by ECI under the Both the river originates in MP and are the powers conferred on it by the tributaries of Yamuna. Representation of the People Act, 1951. Rajghat, Paricha and Matatila dams are over 2. They work under the superintendence Betwa river. and control of the concerned State Ken river passes through Panna tiger reserve. governments.

www.iasgatewayy.com

UPSC NOVEMBER- 2019 DAILY CURRENT AFFAIRS CLASS TEST - 25 3. They help in ensuring the conduct of 5. Which among the following island(s) is/are free and fair polls found in the South China Sea? Which of the statement(s) given above is/are 1. Paracel Islands correct? 2. Spratly Islands (a) 1 and 2 only 3. Kuril Islands (b) 2 and 3 only Choose the correct option (c) 1 and 3 only (a) 1 and 2 only (d) 1, 2 and 3 (b) 2 only Answer: c) (c) 3 only Explanation: (d) None of the above Observers of the Election Commission of Answer: a) India (ECI) are appointed under the powers Explanation: conferred on it by Section 20B of the The Kuril Islands is a chain of islands that Representation of the People Act, 1951 and stretch between the Japanese island of the plenary powers available to the Hokkaido at the southern end and the Commission under the Constitution of India. Russian Kamchatka Peninsula at the They are the appointees of the Commission northern end. working under the superintendence and The islands separate the Sea of Okhotsk from control of the Commission for the period the Pacific Ocean. from their appointment until the process of The island is an issue of contention between election is completed. Russia and Japan. The Section 20B of RPA,1951 provides statutory powers to the Observers to watch 6. Consider the following statements the conduct of elections and especially in regarding World Anti-Doping Agency respect of counting of votes. 1. It is an international organization under The General and Police Observers assist the United Nations Organization. Commission in the conduct of free and fair 2. It aims at harmonizing anti-doping polls. regulations in all sports and countries. Central Observers thus act as an essential link 3. It is collective initiative led by the between the ECI and the booth-level officers International Olympic Committee. (BLO) Which of the statement(s) given above is/are correct? (a) 1 and 2 only (b) 2 and 3 only (c) 1 and 3 only (d) 1, 2 and 3

www.iasgatewayy.com

UPSC NOVEMBER- 2019 DAILY CURRENT AFFAIRS CLASS TEST - 25 Answer: b) 8. Consider the following about Osmosis and Explanation: Reverse Osmosis? It is an International Non-Governmental 1. A process by which molecules of a Organization. solvent tend to pass through a semi It aims at harmonizing anti-doping permeable membrane from a less regulations in all sports and countries. concentrated solution into a more It is collective initiative led by the concentrated one is known as Osmosis. International Olympic Committee. 2. A process by which a solvent pass It was established in 1999 in Lausanne through a porous membrane in the Switzerland under so called Declaration of direction opposite to that for natural Lausanne. osmosis when subjected to a hydrostatic Its headquarters is in Montreal, Canada. pressure greater than the osmotic It is mandated to coordinate, promote and pressure is known as Reverse Osmosis. monitor the fight against drugs in sports. Which of the above are incorrect?

(a) 1 only 7. The famous Living Root Bridges Riwai Root (b) 2 only Bridge and Umshiang Double Decker (c) Both 1 and 2 Bridge are found in (d) Neither 1 nor 2 (a) Meghalaya Answer: d) (b) Manipur Explanation: Both are correct (c) Arunachal Pradesh Why in News: The Supreme Court has (d) Kerala refused to stay the May 2019 order of the Answer: a) National Green Tribunal (NGT) that banned Explanation: the use of reverse osmosis (RO) systems They have been serving as connectors for where drinking water supply had total generations in the Indian state of Meghalaya. dissolved solids (TDS) less than 500 mg per The bridges are primarily a means to cross litre. streams and rivers. They have also become

world-famous tourist attractions. The two 9. Consider the following statements most popular tourist spots are- Riwai Root 1. The Swacch Bharat Mission declared Bridge and Umshiang Double Decker Bridge. India open defecation free on October 2,

2019

2. National Statistical Office (NSO) is not

entitled to conduct survey on sanitation

Which of the above are correct?

www.iasgatewayy.com

UPSC NOVEMBER- 2019 DAILY CURRENT AFFAIRS CLASS TEST - 25 (a) 1 only wrong. But even if you had known that, you’d (b) 2 only be still stuck between B and C. So, entire (c) Both 1 and 2 question hangs on whether you know about (d) Neither 1 nor 2 CRISPR or not. Answer: a) Explanation: The latest National Statistical Office (NSO) survey on sanitation debunked the claims of an open defecation-free or ODF India made by the Centre’s flagship Swachh Bharat scheme.

10. Consider the following pairs: Terms sometimes Context/Topic seen in news 1. Belle II experiment - Artificial Intelligence 2. Block chain technology Digital/Cryptocurrency 3. CRISPR – Cas9 - Particle Physics Which of the pairs given above is/are correctly matched? (a) 1 and 3 only (b) 2 only (c) 2 and 3 only (d) 1, 2 and 3 Answer: b) Explanation: CRISPR, a new genome editing tool, could transform the field of biology. It allows scientists to edit genomes with unprecedented precision. So, #3 is wrong, CRISPR is definitely not associated with Particle Physics. This eliminates A, C and D. Thus, we are left with answer “B: only 2” Belle-II experiment was conducted in Tsukuba, Japan to study violations of the Standard Model of particle physics. So, #1 is

www.iasgatewayy.com

UPSC NOVEMBER- 2019 DAILY CURRENT AFFAIRS CLASS TEST - 26 1. ‘Assamese Gamosa’ which was recently • Paraquat also figures on the list of 99 associated with GI Tag is a pesticides and herbicides the Supreme (a) Sweet made of jaggery Court to ban in an ongoing case. (b) White cotton towel with red border • Paraquat dichloride is being used for 25 (c) Rice variety crops in India, whereas it is approved to be (d) Handmade linen bags used on only nine crops by the Central Answer: b) Insecticide Board and Registration Explanation: Committee. This is a violation of the Indian • Few cultural symbols are as utilitarian as Insecticides Act. the white handmade cotton Gamosa, with • So far in India, only Kerala has banned the its characteristic red border of woven herbicide. motifs. • Another violation: since farmers can’t and • It is also valued as a gift for visitors, used as don’t read the label on paraquat containers, a scarf, anti-dust mask, wrapped around retailers sell paraquat in plastic carry bags the head as a turban. and refill bottles. • Conservationists are now banking on this • Paraquat poisoning, specifically suicide cultural icon to carry forward the message attempts by consuming the fatal chemical, of turtle conservation, with Gamosa woven has emerged as a social tragedy in Odisha. with turtle images. • Unlike other pesticides, insecticides or herbicides, there is no antidote to this 2. Paraquat dichloride, recently seen in news compound. refers to • There are reasons the government has not (a) Toxic pesticide imposed an outright ban on the herbicide. (b) Antibiotic • It has its benefits, like saving farmers (c) Poisonous Gas money and time as it is cheap and (d) Explosive effectively kills weeds quicker than manual Answer: a) de-weeding. Explanation: • Yet, the government could have done more • Paraquat is a toxic chemical that is widely by imposing strict regulations on stock and used as an herbicide (plant killer), sale of the herbicide. Unless open primarily for weed and grass control. availability is curbed, no exercise will be • It has been banned in 32 countries successful in preventing deaths. including Switzerland, where herbicide producing company Sygenta is based.

www.iasgatewayy.com

UPSC NOVEMBER- 2019 DAILY CURRENT AFFAIRS CLASS TEST - 26 3. Bougainville Island sometimes seen in news Explanation: is located in which of the following seas? It is naturally high in vitamin A or beta- (a) Bismarck Sea carotene. (b) Solomon Sea 6. Consider the following about National (c) Coral Sea Sample Survey Organisation - NSSO (d) Timor Sea 1. It conducts household surveys, Answer: b) Explanation: enterprise surveys, village facilities, • It is the eastern most island of Papua New Land & Livestock holdings Guinea, in the Solomon Sea southwester 2. Among the 10-year cycle, Social Pacific. Consumption is taken twice whereas • Along with Buka Island and several island Land & Livestock holdings are taken groups, it forms the autonomous region of only once Bougainville. Which of the above statement(s) is/are • Geographically it is the largest of the incorrect? Solomon Islands. (a) 1 only (b) 2 only 4. Which of the below are powered by same (c) Both 1 and 2 theoretical foundations, as the quantum (d) Neither 1 nor 2 computers Answer: d) Explanation: Both are correct statements 1. Lasers Ten Year Cycle 2. MRI scanners • Consumer Expenditure and Employment & 3. GPS Unemployment - Twice Select the code using the options below: • Social Consumption (health, education etc.) (a) 1 and 2 only - Twice (b) 2 and 3 only • Un-organised Manufacturing - Twice (c) 1, 2 and 3 only (d) 1 and 3 only • Un-organised services - Twice Answer: c) • Land & Livestock holdings - Once Explanation: Factual options. • Open Round - Once • (Special surveys are also undertaken) 5. Golden Rice sometimes seen in news is rich • Annual Consumer Expenditure and in which of the following Vitamins? Employment & Unemployment Surveys (a) Vitamin A (b) Vitamin B (thin sample) (c) Vitamin C (d) Vitamin D Answer: a)

www.iasgatewayy.com

UPSC NOVEMBER- 2019 DAILY CURRENT AFFAIRS CLASS TEST - 26 7. Consider the following statements about 8. Consider the following statements Code on Wages, 2019 regarding ‘Rajya Sabha-Marshals’ 1. The central government will fix a floor 1. Rajya Sabha Marshals are Officers of wage, taking into account living gazetted rank. standards of workers. 2. The President after consultation with 2. The wage fixing entity may set different the Chairman of the Rajya Sabha, make floor wages for different geographical rules regulating the recruitment and the areas. conditions of service of Marshals. 3. The minimum wages decided by the 3. Parliament may by law regulate the central or state governments must be recruitment, and the conditions of higher than the floor wage. service of Marshals. 4. The wage period will be fixed by the Which of the statement(s) given above is/are respective state governments incorrect? Which of the above statements are correct? (a) 1 only (a) 1 and 2 only (b) 1 and 2 only (b) 1,2 and 3 only (c) 1, 2 and 3 (c) 2,3 and 4 only (d) None of the above (d) 3 and 4 only Answer: d) Answer: b) Explanation: Explanation: Payment of wages: Wages will Rajya Sabha Marshals are gazetted officers be paid in Article 98 in The Constitution of India 1949 (i) Coins Secretariat of Parliament (ii) Currency Notes 1. Each House of Parliament shall have a (iii) By Cheque separate secretariat staff: Provided that (iv) By Crediting to the Bank Account, or nothing in this clause shall be construed (v) Through electronic mode. as preventing the creation of posts The wage period will be fixed by the employer common to both Houses of Parliament as either: 2. Parliament may by law regulate the (i) Daily recruitment, and the conditions of service (ii) Weekly of persons appointed, to the secretarial (iii) Fortnightly, or staff of either House of Parliamen (iv) Monthly. 3. Until provision is made by Parliament under clause ( 2 ), the President may, after consultation with the Speaker of the House of the People or the chairman of the council of States, as the case may be, www.iasgatewayy.com

UPSC NOVEMBER- 2019 DAILY CURRENT AFFAIRS CLASS TEST - 26 make rules regulating the recruitment, • With regard to moving amendments to a and the conditions of service of persons Bill, the permission of the Chair is appointed, to the secretarial staff of the required. House of the People or the council of • As regards regulating discussions in the States, and any rules so made shall have House, it is the Speaker who decides as to effect subject to the provisions of any law when a member shall speak and how long made under the said clause Conduct of she shall speak. Business. • If she is satisfied, the Speaker can direct a member to withdraw from the House for a 9. Consider the following statements specific period of time. regarding powers of ‘Speaker of Lok Sabha’ • The Speaker is the guardian of the rights 1. The Speaker is the guardian of the rights and privileges of the House, its and privileges of the House, its Committees and members. Committees and members. • The entire Parliamentary Estate is under 2. The entire Parliamentary Estate is the authority of the Speaker. under the authority of the Speaker. • The Speaker also has certain residuary 3. The Speaker can direct a member to powers under the Rules of Procedure. In withdraw from the House for a specific exercise of this power and under her period of time. inherent powers, the Speaker issues from Which of the statement(s) given above is/are time to time directions which are correct? generally treated as sacrosanct as the (a) 1 and 2 only Rules of Procedure. (b) 2 and 3 only • On matters regarding interpretation of (c) 1, 2 and 3 constitutional provisions relating to the (d) 1 and 3 only House or the Rules of Procedure, she Answer: c) often gives rulings which are respected by Explanation: members and are binding in nature. • Powers of the Speaker - Regulating the • It is the Speaker of the Lok Sabha who Business of the House presides over joint sittings called in the • The Presiding Officers in the two Houses event of disagreement between the two are given vast powers by the rules. Houses on a legislative measure. • It is the Presiding Officer who decides • the admissibility of a Question, • the forms in which amendments may be moved to the Motion of Thanks to the President's Address. www.iasgatewayy.com

UPSC NOVEMBER- 2019 DAILY CURRENT AFFAIRS CLASS TEST - 26 10. Consider the following statements about ‘Lymphatic Filariasis’ 1. It is commonly called as Leishmaniasis. 2. It is a neglected tropical disease caused by parasitic worms. 3. This is a vector-borne disease that spreads through mosquitoes. Which of the statement(s) given above is/are correct? (a) 2 and 3 only (b) 2 only (c) 1 and 3 only (d) 1, 2 and 3 Answer: a) Explanation: • It is commonly called as elephantiasis • It is caused by infection with parasitic worms. • The parasite in its larval stages circulate in the blood and transmitted from person to person by mosquitoes. • It leads to abnormal enlargement of body parts, leading to severe disability. • The affected persons face social stigma and social exclusion. • Kala- Azar is also called as Leishmaniasis

www.iasgatewayy.com

UPSC NOVEMBER- 2019 DAILY CURRENT AFFAIRS CLASS TEST - 27 1. Consider the following pairs: prevent soil erosion and wastage of runoff Traditional Water during periods of heavy rainfall, and also Harvesting Systems States recharge the groundwater. 1. Zing Nagaland Zabo or Ruza System– It is practiced in 2. Kuhls Himachal Pradesh Nagaland. Rainwater that falls on forested 3. Jack wells Kerala hilltops is collected by channels that deposit Which of the pairs given above is/are correct? the run-off water in pond-like structures (a) 1 only created on the terraced hillsides. (b) 2 only Phad – It is a community-managed irrigation (c) 1 and 2 only system in the tapi river basin in Maharashtra. (d) 1, 2 and 3 It starts with check dam built across a river Answer: b) and canals to carry water to agricultural Explanation: blocks with outlets to ensure excess water is Zing – It is found in Ladakh, are small tanks removed from the canals. that collect melting glacier water. A network Panam keni – The Kuruma tribe (a native of guiding channels brings water from the tribe of Wayanad) uses wooden cylinders as a glacier to the tank. special type of well, which are made by Kuhls – They are surface water channels soaking the stems of toddy palms and found in the mountainous regions of immersed in groundwater springs. Himachal Pradesh. The channels carry glacial Ahar Pynes – They are traditional floodwater waters from rivers and streams into the harvesting systems indigenous to South fields. Bihar. Ahars are reservoirs with Jack wells - The Shompen tribe of the Great embankments on three sides and Pynes are Nicobar Islands uses this system, in which artificial rivulets led off from rivers to collect bamboos are placed under trees to collect water in the Ahars for irrigation in the dry runoff water from leaves and carries it to jack months. wells which pits encircled by bunds are made Jhalara - Jhalaras are typically rectangular- from logs of hard wood. shaped step wells that have tiered steps on Pat system – It is developed in Madhya three or four sides in the city of Jodhpur. Pradesh, in which the water is diverted from Bawari - Bawaris are unique step wells that hill streams into irrigation channels by were once a part of the ancient networks of diversion bunds. They are made across the water storage in the cities of Rajasthan. stream by piling up stones and teak leaves Taanka - It is a cylindrical paved and mud. underground pit into which rainwater from Eri – It is tank system, widely used in Tamil rooftops, courtyards or artificially prepared Nadu which acts as flood-control systems,

www.iasgatewayy.com

UPSC NOVEMBER- 2019 DAILY CURRENT AFFAIRS CLASS TEST - 27 catchments flows. It is indigenous to the Thar or an eminent person with special Desert region of Rajasthan. knowledge on Anti-corruption policy. Khadin – Also called dhora, is a long earthen 3. The term of office for Lokpal Chairman embankment that is built across the hill and Members is 5 years or till the age of slopes of gravelly uplands. It is indigenous to 70 years. Jaisalmer region and similar to the irrigation Which of the statement(s) given above is/are methods of Ur region (Present Iraq). correct? Kund – It is a saucer-shaped catchment area (a) 1 only that gently slopes towards the central circular (b) 1 and 2 only underground well. It is found in the sandier (c) 1 and 3 only tracts of western Rajasthan and Gujarat. (d) 1, 2 and 3 Answer: d) 2. Consider the following statements. Explanation: 1. Legal Metrology Act, 2009 aims to A new motto, “Ma Gridhah Kasyasvidhanam establish and enforce standards of (Do not be greedy for anyone’s wealth)”, was weights and measures in India. also adopted. 2. It is administered under Ministry of The Lokpal and Lokayukta Act, 2013 provides Consumer Affairs. the establishment of Lokpal for the Union Which of the above statements is/are and Lokayukta for States. incorrect? These institutions are statutory bodies (a) 1 only without any constitutional status. (b) 2 only They perform the function of an (c) Neither 1 nor 2 "ombudsman” and inquire into allegations of (d) Both 1 and 2 corruption against certain public Answer: c) functionaries and for related matters. Explanation: Both are correct Statements Lokpal is a multi-member body that consists of one chairperson and a maximum of 8 3. Consider the following regarding the members. Lokpal: Chairperson of the Lokpal should be either 1. Lokpal is a statutory body without any the former Chief Justice of India or the constitutional status. former Judge of Supreme Court or an 2. Chairperson of the Lokpal should be eminent person with impeccable integrity and either the former Chief Justice of India outstanding ability, having special knowledge and expertise of minimum 25 years in the or the former Judge of Supreme Court matters relating to anti-corruption policy, public administration, vigilance, finance

www.iasgatewayy.com

UPSC NOVEMBER- 2019 DAILY CURRENT AFFAIRS CLASS TEST - 27 including insurance and banking, law and 4. Consider the following statements management. regarding Protem Speaker in Loksabha . Out of the maximum eight members, half will 1. He is appointed by the President be judicial members and minimum 50% of 2. The main duty of the pro-tem speaker is the Members will be from SC/ ST/ OBC/ to administer the oath of office to new Minorities and women. members of the house. The judicial member of the Lokpal should be 3. Once the new speaker is elected, the either a former Judge of the Supreme Court office of the pro-tem speaker ceases to or a former Chief Justice of a High Court. exist The non-judicial member should be an Which of the above statements is/are correct? eminent person with impeccable integrity and (a) 1 and 2 only outstanding ability, having special knowledge (b) 2 and 3 only and expertise of minimum 25 years in the (c) 1 and 3 only matters relating to anti-corruption policy, (d) All the above public administration, vigilance, finance Answer: d) including insurance and banking, law and Explanation: All the statements are correct management. regarding Protem Speaker The term of office for Lokpal Chairman and

Members is 5 years or till the age of 70 years. 5. What is the correct sequence of occurrence The members are appointed by the president of the following countries as one proceeds on the recommendation of a Selection from north to south? Committee. 1. Montenegro The selection committee is composed of the 2. Albania Prime Minister who is the Chairperson; 3. Greece Speaker of Lok Sabha, Leader of Opposition Select the correct answer using the code given in Lok Sabha, Chief Justice of India or a below. Judge nominated by him/her and One (a) 1-2-3 eminent jurist. (b) 2-1-3 For selecting the chairperson and the (c) 2-3-1 members, the selection committee constitutes (d) 1-3-2 a search panel of at least eight persons. Answer: a)

www.iasgatewayy.com

UPSC NOVEMBER- 2019 DAILY CURRENT AFFAIRS CLASS TEST - 27 6. The Global Economic Prospects report is person in employment matters, released by including recruitment, and promotion. (a) World Bank Which of the statements given above are (b) WEF correct? (c) IMF (a) 1 and 2 only (d) UNCTAD (b) 1 and 3 only Answer: a) (c) 2 and 3 only Explanation: (d) All the above The Global Economic Prospects report is Answer: d) released by World Bank Explanation: The Bill defines a transgender person as one 7. Consider the following Rivers: whose gender does not match the gender 1. Narmada assigned at birth. It includes trans-men and 2. Mandovi trans-women, persons with intersex 3. Daman Ganga variations, gender-queers, and persons with Which of the above flows / flow through Diu socio-cultural identities, such as kinnar and and Daman? hijra. (a) 2 only It prohibits the discrimination against a (b) 3 only transgender person, including denial of (c) 2 and 3 only service or unfair treatment in relation to (d) 1 and 3 only education, employment, healthcare, access to, Answer: b) or enjoyment of goods, facilities, opportunities available to the public. 8. Consider the following statements with Every transgender person shall have a right regard to Transgender Persons (Protection to reside and be included in his household. of Rights) Bill, 2019: The government must provide health 1. It defines a transgender person as one facilities to transgender persons including whose gender does not match the separate HIV surveillance centers, and sex gender assigned at birth. reassignment surgeries. 2. It includes trans-men and trans-women, A transgender person may make an persons with intersex variations, application to the District Magistrate for a gender-queers, and persons with socio- certificate of identity, indicating the gender as cultural identities. ‘transgender’. 3. No government or private entity can The Bill imposes penalties for the offences against transgender persons like bonded discriminate against a transgender labour, denial of use of public places, removal

www.iasgatewayy.com

UPSC NOVEMBER- 2019 DAILY CURRENT AFFAIRS CLASS TEST - 27 from household & village and physical, for governments, private companies, and sexual, verbal, emotional or economic abuse. individuals and civil society to endorse Governments will “Ensure everyone can 9. Consider the following statements with connect to the Internet”, “Keep all of the regard to Contract for the Web: Internet available, all of the time”, and 1. The ‘Contract for the Web’ is a legal “Respect and protect people’s fundamental document. online privacy and data rights”. 2. The idea is to create a global plan of Companies will “Make the Internet affordable action for all stakeholders to together and accessible to everyone”, “Respect and commit to building a “better” Web. protect people’s privacy and personal data to build online trust”, and “Develop 3. The World Wide Web Foundation, a technologies that support the best in non-profit organization founded by Tim humanity and challenge the worst”. Berners-Lee worked on it with The ‘Contract for the Web’ is not a legal Representatives from over 80 document, or a United Nations document — organisations. though the organisations is in talks with the Which of the statements given above are UN. It cannot currently bend governments or correct? companies — even those that are on board — (a) 1 and 2 only to its will. (b) 1 and 3 only

(c) 2 and 3 only 10. Consider the following statements (d) All the above regarding Agni II: Answer: c) 1. It is a surface-to-surface medium range Explanation: nuclear-capable missile. The idea is to create a global plan of action for 2. It is developed by the Defence Research all stakeholders to together commit to and Development Organisation building a “better” Web. (DRDO). The World Wide Web Foundation, a non- 3. It comes under the Integrated Guided profit organization founded by Tim Berners- Missile Development Programme Lee worked on it with Representatives from (IGMDP). over 80 organisations, including Which of the statement(s) given above is/are governments of France, Germany, correct? Switzerland, Italy, and Ghana; tech majors (a) 1 only Google, Facebook, Twitter, etc, civil society (b) 1 and 2 only activists, and academics. The Contract (c) 1 and 3 only consists of following 9 principles, three each (d) 1, 2 and 3

www.iasgatewayy.com

UPSC NOVEMBER- 2019 DAILY CURRENT AFFAIRS CLASS TEST - 27 Answer: d) Explanation: India recently conducted the first, night trial of Agni-II successfully from the Abdul Kalam Island off the Odisha coast. It is a surface-to-surface medium range nuclear-capable missile. Agni-II, an intermediate range (IRBM), has already been inducted into the armed forces. Agni-II was developed by Advanced Systems Laboratory along with other DRDO laboratories and integrated by the . Under Integrated Guided Missile Development Programme (IGMDP) there are five Missiles. They are: 1. Short-range surface-to-surface ballistic missile . 2. Intermediate-range surface-to-surface ballistic missile Agni. 3. Short-range low-level surface-to-air missile . 4. Medium-range surface-to-air missile . 5. Third generation anti-tank missile .

www.iasgatewayy.com

UPSC NOVEMBER- 2019 DAILY CURRENT AFFAIRS CLASS TEST - 28 1. ‘Trachischium apteii’ a new snake species • The dorsal colour of the holotype is dark was recently discovered in brown to black with faint dorsal (a) Kerala longitudinal lines. (b) Maharashtra • Large-sized members of the genus (c) Arunachal Pradesh measure about 293 mm to 299 mm (d) Mizoram (measuring less than a foot that is 300 Answer: c) mm or 30 cm). Explanation: • Trachischium apteii is a non-venomous 2. Consider the following statements burrowing snake in Arunachal Pradesh regarding the discretionary powers of the found under fallen logs inside a thickly Governor: forested area of the Tally Valley Wildlife 1. If there is NO political party with a clear- Sanctuary near the town of Ziro in cut majority in the assembly, Governor on Arunachal Pradesh. his/ her discretion can appoint anybody • The newly discovered species belongs to a as chief minister. group of fossorial snakes that live mostly 2. On his/ her discretion, the Governor can underground, and surface mainly during reserve a bill passed by the state or after a heavy monsoon shower. legislature for president’s assent. • Experts behind the discovery suggested 3. Governor, on his/ her discretion can that due to the burrowing habits of recommend the president about the species of this genus, snakes belonging to failure of the constitutional machinery in the group are seldom seen and hence the state. remain poorly studied. 4. It is up to the Governor what he/ she • Trachischium apteii was named so to would like to do when the he legislative honour the contribution of ‘Deepak Apte’, assembly is dissolved following a vote of well-known marine biologist and Director no confidence. of the (Bombay Natural History Society) Which of the statements given above are BNHS. correct? • Trachischium species are commonly (a) 1 and 2 only called slender snakes, and are currently (b) 2 and 4 only known by seven species that are (c) 1, 2 and 3 only distributed across the Himalayas, and the (d) All the above Indo-Burma and Indo-China regions. Answer: d) • Morphologically, the snake is Explanation: distinguished by smooth and dorsal scales • Some discretionary powers of the arranged in 15 rows throughout the body. Governor are as follows:

www.iasgatewayy.com

UPSC NOVEMBER- 2019 DAILY CURRENT AFFAIRS CLASS TEST - 28 • Governor can dissolve the legislative 3. Consider the following statements assembly if the chief minister advices him regarding the Dadra and Nagar Haveli and to do following a vote of no confidence. Daman and Diu (Merger of Union Now, it is up to the Governor what he/ she Territories) Bill, 2019: would like to do. 1. The Bill seeks to amend the Schedule to • Governor, on his/ her discretion can allocate two Lok Sabha seats to the recommend the president about the merged UT. failure of the constitutional machinery in 2. The Bill provides that the jurisdiction of the state. the High Court of Bombay will continue to • On his/ her discretion, the Governor can extend to the merged UT. reserve a bill passed by the state Which of the statements given above is/are legislature for president’s assent. correct? • If there is NO political party with a clear- (a) 1 only cut majority in the assembly, Governor on (b) 2 only his/ her discretion can appoint anybody (c) Both 1 and 2 as chief minister. (d) Neither 1 nor 2 • Governor determines the amount payable Answer: c) by the Government of Assam, Meghalaya, Explanation: Tripura and Mizoram to an autonomous • The Bill amends the First Schedule to Tribal District Council as royalty accruing merge the territories of the two UTs: (a) from licenses for mineral exploration. Dadra and Nagar Haveli, and (b) Daman • Governor can seek information from the and Diu. chief minister with regard to the • The merged territory will form the single administrative and legislative matters of UT and named as Dadra and Nagar Haveli the state. and Daman and Diu. • Governor has discretion to refuse to sign • Article 240(1) of the Constitution allows to an ordinary bill passed by the state the President to make regulations for legislature. certain UTs, including the UTs of Dadra • Thus, though the Governor is made the and Nagar Haveli, and Daman and Diu. constitutional head of a state like • The Dadra and Nagar Haveli and Daman president of India, yet there is a thin line and Diu (Merger of Union Territories) as the Constitution empowers the Bill, 2019 will help in strengthening Governor to act without the advice of the administrative efficiency services. Chief Minister and his council and can use • The First Schedule to the Representation discretion on certain matters. of the People Act, 1950 provides one seat in Lok Sabha to each of the two UTs. The www.iasgatewayy.com

UPSC NOVEMBER- 2019 DAILY CURRENT AFFAIRS CLASS TEST - 28 Bill seeks to amend the Schedule to between India and EU, which was allocate two Lok Sabha seats to the initiated in 2007. merged UT. • Even after a decade of negotiations, India • Provisions retained are reservation and EU have failed to resolve certain provided to people in the two union issues which have led to a deadlock. territories will continue, the number of • Important Points Lok Sabha seats will also remain • India has not been granted “data secure” unaltered and the jurisdiction of the status by EU. Bombay High Court will continue over the • India does not want an FTA without two territories, there will be no change in investment being part of it. the status of Group III and IV employees. • U.K. visa rules discriminate against • The country currently has nine Union Indian technical professionals including Territories after the creation of the UTs of because they have hiked visa fees and Jammu and Kashmir, and Ladakh. have numerical caps on visas. However, with the merger of Daman and • EU imposed a ban on sale of 700 Diu, and Dadra and Nagar Haveli, the pharmaceutical products even though number of UTs will come down to eight. they were clinically tested by GVK Biosciences. 4. With reference to Bilateral Trade and • India cancelled most bilateral investment Investment Agreement (BTIA), Consider agreements with EU member states in the following statements: 2016 on grounds that they were outdated. 1. It is signed between India and European • Presence of non-tariff barriers on Indian Union in 2016. agricultural products in the form of 2. It covers Trade in Goods, Trade in sanitary and phytosanitary measures Services, Investment, Sanitary and which are too stringent and enable the EU Phytosanitary Measures. to bar many Indian agricultural products Which of the statement(s) given above is/are from entering its markets. incorrect?

(a) 1 only 5. Consider the following with regard to (b) 2 only characteristics of Madden Julian Oscillation (c) Both 1 and 2 (MJO): (d) Neither 1 and 2 1. It is a movement of rain clouds westward Answer: a) over the tropics. Explanation: 2. It regulates tropical cyclone, monsoons • Bilateral Trade and Investment and El Nino cycle. Agreement is a Free Trade Agreement

www.iasgatewayy.com

UPSC NOVEMBER- 2019 DAILY CURRENT AFFAIRS CLASS TEST - 28 3. It has two phases; they are Convective entire dipole that is having two main rainfall phase and Suppressed rainfall opposing centers of action propagates phase. eastward. Which of the statements given above are • The location of the convective phases is correct? often grouped into geographically based (a) 1 and 2 only stages that climate scientists (b) 1 and 3 only (c) 2 and 3 only 6. Which one of the following statements (d) All the above appropriately describes the term Answer: c) ‘recession’? Explanation: 1. It is the drastic deficiency in supply of • It is a Phenomenon, that have it can have goods, which results in inflation. dramatic impacts in the mid-latitudes. 2. It refers to a significant decline in • Several times a year the MJO is a strong economic activity and is spread across contributor to various extreme events in the economy. the United States, including Arctic air 3. It refers to the decrease in the foreign outbreaks during the winter months investment leading to unemployment. across the central and eastern portions of 4. It refers to the depreciation in the the United States. monetary value of the country’s • It is characterized by a band of rain clouds currency. moving eastward over the tropics. Answer: b) • It regulates tropical cyclone, the Explanation: monsoons and the El Nino cycle and • A recession is a significant decline in contributes to severe weather events over economic activity spread across the Asia, Australia, Africa, Europe and economy, lasting more than a few America. months, normally visible in real GDP, • It consists of two parts, or phases: one is real income, employment, industrial the enhanced rainfall or convective phase production, and wholesale-retail sales. and the other is the suppressed rainfall • A recession begins just after the phase. economy reaches a peak of activity and • Strong MJO activity often dissects the ends as the economy reaches its planet into halves: one half within the trough. enhanced convective phase and the other • Depression is nothing but a severe half in the suppressed convective phase. form of recession. • These two phases produce opposite

changes in clouds and rainfall and this

www.iasgatewayy.com

UPSC NOVEMBER- 2019 DAILY CURRENT AFFAIRS CLASS TEST - 28 1949, and he continued to occupy the 7. Which among the following island(s) is/are office of the Speaker till the First Lok located in the Mediterranean Sea Sabha was constituted in 1952. 1. Sicily • G V Mavalankar was the first Speaker of 2. Crete Lok Sabha. 3. Malta • The rules formulated by him to recognize 4. Canary the leader of opposition in the house have (a) 1 and 2 only widely come to be known as (b) 1, 2, and 3 only Mavalankar’s rule. (c) 2, 3 and 4 only • Under Mavalankar’s rule, the single (d) 1, 2, 3 and 4 largest opposition party must have a Answer: b) strength of at least 10 per cent of the total Explanation: strength of the House in order to Canary Island- is located in Eastern designate its leader as the Leader of Atlantic Ocean. Opposition.

• The Supreme Court in its judgement, has 8. Consider the following statements stated that Mavalankar’s rule was not a regarding G.V. Mavalankar statutory provision and further Speaker’s 1. He served as the Speaker of the statement or procedure evolved to run the Provisional Parliament from 1949 till House was outside the purview of judicial 1952. review. 2. He served as the president of INC when • In the 17th Lok Sabha, there were no party Netaji Subhash Chandra Bose resigned which can qualify for the post of leader of in 1939. opposition based on the numerical 3. He served as the Speaker of the First strength as provided under the Lok Sabha Mavalankar’s rule. Which of the statements given above is/are • Statement 2 is wrong - Rajendra Prasad correct? became the president when Netaji (a) 1 only Subhash Chandra Bose resigned in 1939. (b) 2 and 3 only (c) 1 and 3 only (d) 1, 2 and 3 Answer: c) Explanation: • Mavalankar became the Speaker of the Provisional Parliament on 26 November

www.iasgatewayy.com

UPSC NOVEMBER- 2019 DAILY CURRENT AFFAIRS CLASS TEST - 28

9. Consider the following statement regarding Explanation: the Polar Satellite Launch Vehicle (PSLV): • The International Solar Alliance (ISA) is 1. It is the first Indian launch vehicle to be an alliance of more than 122 countries equipped with liquid stages. jointly initiated by India and France most 2. It is designed mainly to deliver the of them being sunshine countries, which lie either completely or partly between the “earth-observation” or “remote-sensing” Tropic of Cancer and the Tropic of satellites. Capricorn, now extended to all members Which of the statement(s) given above is/are correct? of UN. (a) 1 only (b) 2 only (c) Both 1 and 2 (d) Neither 1 nor 2 Answer: c) Explanation: • Polar Satellite Launch Vehicle (PSLV) is the third generation launch vehicle of India. It is the first Indian launch vehicle to be equipped with liquid stages. • PSLV is used for delivering various satellites to Low Earth Orbits. It is designed mainly to deliver the “earth- observation” or “remote-sensing” satellites. With lift-off mass of up to about 1750 Kg to Sun-Synchronous circular polar orbits of 600-900 Km altitude.

10. International Solar Alliance (ISA) was jointly initiated by which of the following countries? (a) India and USA (b) India and UK (c) India and France (d) India and China Answer: c)

www.iasgatewayy.com

UPSC NOVEMBER- 2019 DAILY CURRENT AFFAIRS CLASS TEST - 29 1. Consider the following statements • NATO has only once invoked Article 5, on regarding the NATO: September 12, 2001 following the 9/11 1. NATO is a military alliance established attacks on the World Trade Center in the by the United States, Canada, and US. several Western European nations to • As of 2019, there are 29 member states, provide collective security against the with Montenegro becoming the latest Soviet Union. member to join the alliance in 2017. 2. Montenegro became the latest member • France withdrew from the integrated to join NATO in 2017. military command of NATO in 1966 but 3. Recently US have withdrawn from the remained a member of the organization. alliance after the dispute regarding However, it resumed its position in NATO’s military command in 2009. operating budget in NATO. • The U.S. has recently announced that it Which of the statement(s) given above would cut its contribution to NATO’s is/are correct? operating budget. (a) 1 only (b) 1 and 2 only 2. Consider the following statements (c) 1 and 3 only regarding the FASTags: (d) 1, 2 and 3 1. The FASTag is a reloadable tag that Answer: b) allows automatic deduction of toll Explanation: without having to stop for carrying out • North Atlantic Treaty Organization the cash transaction. (NATO) is a military alliance established 2. The tag uses radio frequency by the North Atlantic Treaty (also called identification (RFID) technology and is the Washington Treaty) of April 4, 1949, fixed on the windscreen of the vehicle by the United States, Canada, and several once active. Western European nations to provide 3. National Highways Authority of India collective security against the Soviet (NHAI) and National Payment Union. Corporation of India (NPCI) are • A key provision of the treaty, the so-called implementing this program. Article 5, states that if one member of the Which of the statement(s) given above alliance is attacked in Europe or North is/are incorrect? America, it is to be considered an attack on all members. That effectively put (a) 1 only (b) 1 and 3 only Western Europe under the "nuclear (c) 3 only (d) 2 and 3 only umbrella" of the US. Answer: c)

www.iasgatewayy.com

UPSC NOVEMBER- 2019 DAILY CURRENT AFFAIRS CLASS TEST - 29 Explanation: 3. Mission 41K, sometimes seen in news is • The FASTag is a reloadable tag that allows referred to automatic deduction of toll without (a) Training 41,000 physically challenged having to stop for carrying out the cash students across India in the sports field to transaction. revive their livelihoods. • The tag uses radio frequency (b) Saving of Rs. 41,000 Crore through identification (RFID) technology and is integrated Rail Energy Management fixed on the windscreen of the vehicle System by 2025. once active. (c) To bring back the 41,000-crore debt from • Radio-Frequency Identification (RFID) is various Multi-National Companies. the use of radio waves to read and capture (d) Indian Government’s target of 41,000 KW information stored on a tag attached to an Solar power in rural parts of India by object. A tag can be read from up to 2022. several feet away and does not need to be Answer: b) within direct line-of-sight of the reader to Explanation: be tracked. • “Mission 41K” document was unveiled by • The tag is valid for five years and comes in Ministry of Railways in 2017. It targets for seven different colours — violet, orange, saving of Rs. 41,000 Crore through yellow, green, pink, blue, black. Each integrated Rail Energy Management colour is assigned to a particular category System by 2025. of vehicles. • Railway Energy Management Company • To encourage the use of FASTags, the Limited (REMCL), a JV of Ministry of National Highway Authority of India Railways and RITES Ltd., has been (NHAI) refunds 5% of the total monthly endeavouring to achieve this target by transactions. gradual migration from Consumer to • Indian Highways Management Company Deemed Licensee. Limited (IHMCL) (a company • Deemed Licensee status enables Indian incorporated by National Highways Railways to buy electricity directly from Authority of India) and National Payment any generating company by paying Corporation of India (NPCI) are wheeling charges to Central and State implementing this program. transmission system under open access as • FASTag is presently operational at both, per The Electricity Act, 2003. national and state highways. • These efforts have resulted in keeping electricity price at most economical rate on sustainable basis since 2015 on Indian Railways.

www.iasgatewayy.com

UPSC NOVEMBER- 2019 DAILY CURRENT AFFAIRS CLASS TEST - 29 4. Consider the following pairs: 6. LB-1 recently seen in news is referred to Defence system Country (a) Planetary Nebula 1. Arrow missile system - Israel (b) Red dwarf 2. Phalcon - Russia (c) Black hole 3. AAD (Advanced air defence) - India (d) White dwarf Which of the pairs given above are correct? Answer: c) (a) 1 and 2 only Explanation: (b) 2 and 3 only LB-1 (c) 1 and 3 only • The Astronomers discovered a huge Black (d) 1, 2 and 3 hole LB-1 in our Milky Way galaxy. Answer: c) • It is discovered using China’s LAMOST Explanation: telescope. • Phalcon is an airborne warning and • It is 15,00 light year away from earth and control systems (AWACS) developed by mass 70 times greater than the sun. Israel. • According to most of the current models of Steller evolution, Black holes of such 5. Consider the following Tiger Reserves in mass should not exist in our galaxy. India: • Steller black holes are usually formed in 1. Kamlang Tiger Reserve aftermath of supernova explosion, a 2. Nameri Tiger Reserve phenomenon that occurs when extremely 3. Orang Tiger Reserve large star burn out at the end of their 4. Manas Tiger Reserve lives. Which of the following Tiger Reserves are • LB-1’s large mass falls into a range known located in the state of Assam? as ‘pair instability gap’ where supernova (a) 1, 2 and 3 only should not have produced it. (b) 2, 3 and 4 only Planetary Nebulas (c) 1, 3 and 4 only • It is abbreviated as PN or plural PNe, is a (d) All the above type of emission nebula consisting of an Answer: b) expanding, glowing shell of ionized gas Explanation: ejected from red giant stars late in their • Kamlang Tiger Reserve is located in the lives. state of Arunachal Pradesh. White Dwarf • Tiger Reserves in Assam are Manas Tiger • It is also called as degenerate dwarf, is a Reserve, Orang Tiger Reserve, Nameri stellar core remnant composed mostly of Tiger Reserve and Kaziranga Tiger electron-degenerate matter. A white dwarf Reserve. (Hint: MONK). is very dense: its mass is comparable to www.iasgatewayy.com

UPSC NOVEMBER- 2019 DAILY CURRENT AFFAIRS CLASS TEST - 29 that of the Sun, while its volume is under this Act, irrespective of the location comparable to that of Earth. of crime / offense committed. Red Dwarf Which of the statement(s) given above is/are • It is the smallest and coolest kind of star correct? on the main sequence. Red dwarfs are by (a) 1 only far the most common type of star in the (b) 2 only Milky Way, at least in the neighbourhood (c) Both 1 and 2 of the Sun, but because of their low (d) Neither 1 and 2 luminosity, individual red dwarfs cannot Answer: c) be easily observed. Explanation: • It is applicable across the entire country 7. Global Migration Report is released by • Any Indian or foreign national charged (a) Global Forum on Migration and under UAPA is liable for punishment Development under this Act, irrespective of the location (b) International Organisation for Migration of crime / offense committed (c) United Nation High Commission for • UAPA will be applicable to the offenders Refugees in the same manner, even if crime is (d) United Nation Development Program committed on a foreign land, outside Answer: b) India Explanation: • The provisions of this Act apply also to It is released by International Organization of citizens of India and abroad. Migration • Persons on ships and aircrafts, registered It provides an object and balance account of in India, wherever they may be are also migration globally. under the ambit of this act. It can inform our own work, be it in studies, research and analysis, policymaking, 9. Consider the following statements with communication, or migration practice. regard to Exercise Him Vijay 1. It is a bilateral mountain assault exercise 8. Consider the following with regard to between India and Nepal Unlawful Activities Prevention Act 2. The exercise is to test mobility, (UAPA),2019: communication and coordination of such 1. It is applicable across the entire country huge body of fast-moving troops in including the Union Territory of Jammu different terrains. and Kashmir. Which of the statement(s) given above is/are 2. Any Indian or foreign national charged correct? under UAPA is liable for punishment

www.iasgatewayy.com

UPSC NOVEMBER- 2019 DAILY CURRENT AFFAIRS CLASS TEST - 29 (a) 1 only Explanation: (b) 2 only • It is extracted from unconventional gas (c) Both 1 and 2 reservoirs where gas is extracted directly (d) Neither 1 nor 2 from the rock that is the source of the gas. Answer: b) • The methane is held underground within Explanation: the coal and is extracted by drilling into • The Indian Army Conducts routine the coal seam and removing the military exercises every year. groundwater. • It was one such endeavour. It was • The resulting drop in pressure causes the conducted to validate operational methane to be released from the coal. capabilities of our combat formations. CBM has been looked at as a clean • The exercise is to test mobility, alternative fuel with significant prospects. communication and coordination of such CBM can be used for power generation, as huge body of fast-moving troops in compressed natural gas (CNG) auto fuel, difficult terrain. as feedstock for fertilizers, industrial uses • Indian Army is metamorphosing itself to such as in cement production, rolling incorporate changes for modern and mills, steel plants, and for methanol quick and short war, added the Army production. officer. • It is called 'sweet gas’ because of its lack of Hydrogen Sulphide. 10. Consider the following statements with • It often contains few percent of carbon regard to Coalbed methane (CBM): dioxide. 1. It contains huge amount of carbon dioxide. 2. It is used as Compressed natural gas, fertilizers and industrial purpose. 3. It is also known as sweet gas. Which of the statements given above are correct? (a) 1 and 2 only (b) 1 and 3 only (c) 2 and 3 only (d) All the above Answer: c)

www.iasgatewayy.com

UPSC NOVEMBER- 2019 DAILY CURRENT AFFAIRS CLASS TEST - 30 1. ‘Phishing’ which was recently seen in news attackers interrupt the traffic, they can is referred to filter and steal data. (a) It refers to any kind of software that is • SQL Injection: SQL (pronounced designed to cause damage to a single “sequel”) stands for Structured Query computer, server, or computer network. Language, a programming language used (b) It is the method of trying to gather to communicate with databases. personal information using deceptive e- • Many of the servers that store critical data mails and websites. for websites and services use SQL to (c) It is an attack meant to shut down a manage the data in their databases. machine or network, making it • A SQL injection attack specifically targets inaccessible to its intended users such kind of servers, using malicious code (d) It is a programming language used to to get the server to divulge information it communicate with databases. normally wouldn’t. Answer: b) • Cross-Site Scripting (XSS): Similar to Explanation: an SQL injection attack, this attack also • Malware, short for malicious software involves injecting malicious code into a refers to any kind of software that is website, but in this case the website itself designed to cause damage to a single is not being attacked. computer, server, or computer network. • Instead the malicious code the attacker Ransomware, Spy ware, Worms, viruses, has injected, only runs in the user's and Trojans are all varieties of malware. browser when they visit the attacked • Phishing: It is the method of trying to website, and it goes after the visitor gather personal information using directly, not the website. deceptive e-mails and websites. • Social engineering is an attack that relies • Denial of Service attacks: A Denial-of- on human interaction to trick users into Service (DoS) attack is an attack meant to breaking security procedures in order to shut down a machine or network, making gain sensitive information that is typically it inaccessible to its intended users. DoS protected. attacks accomplish this by flooding the target with traffic, or sending it 2. India is a signatory to which of the information that triggers a crash. following Defence agreements of US? • Man-in-the-middle (MitM) attacks, 1. General Security of Military Information also known as eavesdropping attacks, Agreement (GSOMIA) occur when attackers insert themselves 2. Logistics Exchange Memorandum of into a two-party transaction. Once the Agreement (LEMOA)

www.iasgatewayy.com

UPSC NOVEMBER- 2019 DAILY CURRENT AFFAIRS CLASS TEST - 30 3. Communications Compatibility and • BECA will allow India and US militaries Security Agreement (COMCASA) to share geospatial and satellite data with 4. Basic Exchange and Cooperation each other. Agreement (BECA) • India has not signed BECA yet. Select the correct answer using the codes given below: 3. Choose which of the following missiles are (a) 1 and 2 only indigenously developed missiles of India: (b) 1, 2 and 3 only 1. AMOGHA MISSILE (c) 1, 3 and 4 only 2. NAG (d) All the above 3. HELINA Answer: b) Choose the correct option using the code Explanation: given below: • “Four Foundational Agreements” between (a) 1 and 2 only the US and its Partners (b) 2 and 3 only General Security of Military (c) 1 and 3 only Information Agreement (GSOMIA) (d) 1,2 and 3 • GSOMIA allows militaries to share the Answer: d) intelligence gathered by them. Explanation: • Signed by India in 2002. All the above missiles are indigenously Logistics Exchange Memorandum of produced. So, the answer is d. Agreement (LEMOA) • LEMOA allows both countries to have 4. Consider the following statements access to each other’s designated military regarding the codes of Labour laws, which facilities for refuelling and replenishment. is recently in news: • Signed by India in 2016. 1. The government is subsuming a total of Communications and Information 44 labour laws into five codes. Security Memorandum of Agreement 2. Only 2 of the labour codes functions (CISMOA) under the Labour ministry. • COMCASA (Communications Choose the correct answer using the code Compatibility and Security Agreement) is given below: the India specific version of CISMOA. (a) 1 only • Signed by India in 2018. (b) 2 only Basic Exchange and Cooperation (c) Both 1 and 2 Agreement (BECA) (d) Neither 1 nor 2 Answer: d)

www.iasgatewayy.com

UPSC NOVEMBER- 2019 DAILY CURRENT AFFAIRS CLASS TEST - 30 Explanation: Explanation: The 44 laws are subsumed into 4 laws. So, • It is a multi-stakeholder alliance launched statement 1 is incorrect. by UNICEF All the 4 codes function under the labour • It aims to facilitate youth to gain relevant ministry. So, statement 2 is also incorrect. skills for productive lives and the future of work. 5. Consider the following statements: • The targeted age group of YuWaah 1. The 6th Asian Dendrochronology includes adolescent girls and boys. Conference being held at the Birbal Sahni • It intends to create platforms to guide Institute of Palaeo sciences in Lucknow. youth in market opportunities like career 2. Dendrochronology is the study of tree guidance, mentorship, internships, rings that hold a wealth of information. apprenticeships and facilitate integration Which of the statement(s) given above is/are of career guidance in school education. correct? (a) 1 only 7. Consider the following statements (b) 2 only regarding Light-Water Rector (LWR) (c) Both 1 and 2 1. It uses normal water as both coolant (d) Neither 1 and 2 and neutron moderator. Answer: c) 2. It is also a variety of Thermal-Nuclear Explanation: Reactor. • The 6th Asian Dendrochronology 3. India's first light water reactor is the Conference being held at Lucknow. So, Kalpakkam Nuclear Reactor. statement 1 is correct. Which of the statements given above are • It is the study of tree rings that hold a correct? wealth of information about not only a (a) 1 and 2 only tree’s past but also that of the ecosystem (b) 2 and 3 only in which it lives. So, statement 2 is correct (c) 1 and 3 only

(d) All the above 6. YuWaah Initiative, launched by UNICEF Answer: a) which is recently seen in news refers to Explanation: (a) Delivery of vaccines by drone • It is a type of thermal-neutron reactor (b) Innovation for children that uses NORMAL WATER, as opposed (c) Multi-stakeholder alliance to heavy water, as both its coolant and (d) AI Workshop neutron moderator. So, statement 1 is Answer: c) correct.

www.iasgatewayy.com

UPSC NOVEMBER- 2019 DAILY CURRENT AFFAIRS CLASS TEST - 30 • It the most common type of thermal- • The Agency came into existence with the neutron reactor. So, statement 2 is enactment of the National Investigation correct. Agency Act 2008. • Kudankulam Nuclear Power Project • The agency was raised after 2008 (KNPP) is the India's first pressurised Mumbai terror attacks, to create a water reactor belong to the light water deterrent force. reactor. So, statement 3 in incorrect. • It aims to set the standards of excellence in counter terrorism and other national security related investigations at the 8. Consider the following statements national level by developing into a highly regarding National Investigation Agency trained, partnership-oriented workforce. (NIA) • It also aims to develop as a storehouse of 1. It is the Central Counter terrorism Law all terrorist related information. Enforcement Agency. • Various Special Courts have been notified 2. Mulla Committee recommended the by the Central Government of India for establishment of NIA. trial of the cases registered at various 3. NIA can also probe in to violations of police stations of NIA under Section 11 atomic energy act. and 22 of the NIA Act 2008. Which of the statements given above is/are • Statement 2 is wrong- Mulla correct? Committee on Prison reforms (a) 1 and 3 only recommended establishment of National (b) 2 and 3 only Commission on Prisons. (c) 2 only

(d) 1, 2 and 3 9. What is common to the drugs known as Answer: a) isoniazid, rifampicin and fluoroquinolone? Explanation: (a) These are single dose combination drugs • The National Investigation Agency that used in treating Kala-azar. (NIA) is a central agency established by (b) These are antibiotics that are used in the Indian Government to combat terror treating tuberculosis. in India. (c) These drugs are used to treat Alzheimer's • It acts as the Central Counter Terrorism disease in the elderly. Law Enforcement Agency. (d) These are prescription drugs that are • The agency is empowered to deal with misused as stimulants in sports activities. terror related crimes across states without Answer: b) special permission from the states.

www.iasgatewayy.com

UPSC NOVEMBER- 2019 DAILY CURRENT AFFAIRS CLASS TEST - 30 Explanation: Which of the statement(s) given above • Isoniazid, rifampicin and is/are correct? fluoroquinolone are antibiotics that are (a) 1 only used in treating tuberculosis. (b) 2 only • Isoniazid and rifampicin – are the first (c) 1 and 2 only line anti-biotics (d) 1, 2 and 3 • People with TB who do not respond to at Answer: d) least isoniazid and rifampicin, are said to Explanation: have MDR-TB. • The Conference of Parties (COP) is the • People who are resistant to isoniazid and supreme decision-making body of the rifampin, plus any fluoroquinolone and at UNFCCC. least one of three injectable second-line • All States that are Parties to the drugs (amikacin, kanamycin, or Convention are represented at the COP, capreomycin) are said to have XDR-TB. • The Parties review the implementation of • Recently, US-FDA has approved a new the Convention and other legal drug Pretomanid for treating drug- instruments that the COP adopts and take resistant tuberculosis — multidrug- decisions necessary to promote the resistant TB (MDR-TB) and extensively effective implementation of the drug-resistant TB (XDR-TB). Convention.

• Massive protests triggered by social 10. In the context of United Nations unrest over economic issues have forced Framework Convention on Climate Change Chile to cancel its hosting of COP25. (UNFCCC) consider the following • So, the COP25 will take place under the statements: Presidency of the Government of Chile, to 1. Conference of Parties (COP) is the be held in Madrid, Spain. supreme decision-making body of the • The logistical support will be provided by UNFCCC. the Government of Spain. 2. COP25 to UNFCCC will take place under • Unlike its predecessor, the Kyoto the Presidency of the Government of Protocol, which sets commitment targets Chile, to be held in Madrid, Spain. that have legal force, the Paris Agreement, 3. Paris Agreement which was adopted in with its emphasis on consensus-building, COP 21 is voluntary unlike Kyoto allows for voluntary and nationally protocol which is legally binding on determined targets. The specific climate parties. goals are thus politically encouraged, rather than legally bound.

www.iasgatewayy.com